Sunteți pe pagina 1din 261

CBSE

SAMPLE
PAPERS
(March 2018)

X
MATHEMATICS
CONTENTS

1. Sample paper (By CBSE )…….…...………………………………………………..….01-16

2. Sample paper 01………….……………………………………………………………..…17-33

3. Sample paper 02………….……………………………………………………………..…34-51

4. Sample paper 03………….……………………………………………………………..…52-77

5. Sample paper 04………….……………………………………………………………..…78-98

6. Sample paper 05………….……………………………………………………………..…99-118

7. Sample paper 06………….……………………………………………………………..…119-137

8. Sample paper 07………….……………………………………………………………..…138-169

9. Sample paper 08………….……………………………………………………………..…170-190

10. Sample paper 09………….……………………………………………………………..…191-211

11. Sample paper 10………….……………………………………………………………..…212-232

12. Sample paper 11………….……………………………………………………………..…233-259


SAMPLE QUESTION PAPER (By CBSE)
Class-X (2017–18) Mathematics

General Instructions:

1. All questions are compulsory.


2. The question paper consists of 30 questions divided into four sections A, B, C and D.
3. Section A contains 6 questions of 1 mark each. Section B contains 6 questions of 2 marks
each. Section C contains 10 questions of 3 marks each. Section D contains 8 questions of 4
marks each.
4. There is no overall choice. However, an internal choice has been provided in four
questions of 3 marks each and three questions of 4 marks each. You have to attempt only
one of the alternatives in all such questions.
5. Use of calculators is not permitted.

Section A
Question numbers 1 to 6 carry 1 mark each

1. Write whether the rational number will have a terminating decimal expansion or
a nor-terminating repeating decimal expansion.
Ans. Non terminating repeating decimal expansion.
2. Find the value(s) of k, if the quadratic equation has equal roots.
Ans.
3. Find the eleventh term from the last term of the AP:
27, 23, 19, ..., –65.
Ans.
4. Find the coordinates of the point on y-axis which is nearest to the point (–2, 5).
Ans.
5. In given figure, and Find the ratio of the area of
to the area of

Material downloaded from myCBSEguide.com. 1 / 16


Ans.
6. If find the value of
Ans. 25

Section B
Question numbers 7 to 12 carry 2 marks each.

7. If two positive integers p and q are written as are prime


numbers, then verify:

Ans.

8. The sum of first n terms of an AP is given by Find the sixteenth term


of the AP.

Ans. Sn = 2n2 + 3n

S1 = 5 = a1

S2 = a1 + a2 = 14 a2 = 9

d = a2 – a1 = 4

a16 = a1 + 15d = 5 + 15(4) = 65

9. Find the value(s) of k for which the pair of linear equations


have infinitely many solutions.

Ans. For pair of equations kx + 1y = k2 and 1x + ky = 1


We have:

For infinitely many solutions,

Material downloaded from myCBSEguide.com. 2 / 16


From (i) and (ii), k = 1

10. If is the mid-point of the line segment joining the points (2, 0) and

then show that the line passes through the point

Ans. Since is the mid-point of the line segment joining the points

therefore,
The line passes through the point
11. A box contains cards numbered 11 to 123. A card is drawn at random from the box.
Find the probability that the number on the drawn card is
(i) a square number
(ii) a multiple of 7
Ans. (i) P(square number)
(ii) P(multiple of 7)
12. A box contains 12 balls of which some are red in colour. If 6 more red balls are put
in the box and a ball is drawn at random, the probability of drawing a red ball doubles
than what it was before. Find the number of red balls in the bag.
Ans. Let number of red balls be = x

If 6 more red balls are added:


The number of red balls = x + 6

Since,
There are 3 red balls in the bag.

Section C
Question numbers 13 to 22 carry 3 marks each.

13. Show that exactly one of the numbers is divisible by 3.


Ans. Let n = 3k, 3k + 1 or 3k + 2.
(i) When n = 3k:
n is divisible by 3.
n + 2 = 3k + 2 n + 2 is not divisible by 3.
n + 4 = 3k + 4 = 3(k + 1) + 1 n + 4 is not divisible by 3.

Material downloaded from myCBSEguide.com. 3 / 16


(ii) When n = 3k + 1:
n is not divisible by 3.
n + 2 = (3k + 1) + 2 = 3k + 3 = 3(k + 1) n + 2 is divisible by 3.
n + 4 = (3k + 1) + 4 = 3k + 5 = 3(k + 1) + 2 n + 4 is not divisible by 3.
(iii) When n = 3k + 2:
n is not divisible by 3.
n + 2 = (3k + 2) + 2 = 3k + 4 = 3(k + 1) + 1 n + 2 is not divisible by 3.
n + 4 = (3k + 2) + 4 = 3k + 6 = 3(k + 2) n + 4 is divisible by 3.
Hence exactly one of the numbers n, n + 2 or n + 4 is divisible by 3.
14. Find all the zeroes of the polynomial if two of its zeroes

are

Ans. Since are the two zeroes therefore,

is a factor of given polynomial.

We divide the given polynomial by

For other zeroes,

Zeroes of the given polynomial are

15. Seven times a two digit number is equal to four times the number obtained by
reversing the order of its digits. If the difference of the digits is 3, determine the
number.
Ans. Let the ten’s and the units digit be y and x respectively.
So, the number is 10y + x.
The number when digits are reversed is 10x + y.

Material downloaded from myCBSEguide.com. 4 / 16


Now, 7(10y + x) = 4(10x + y) 2y = x …(i)
Also x – y = 3 …(ii)
Solving (1) and (2), we get y = 3 and x = 6.
Hence the number is 36
16. In what ratio does the x-axis divide the line segment joining the points
Find the co-ordinates of the point of division.
OR
The points form a parallelogram. Find the
length of the altitude of the parallelogram on the base AB.
Ans. Let x-axis divides the line segment joining at the point P in the
ratio 1 : k.

Now, coordinates of point of division

Since P lies on x-axis, therefore

Hence the ratio is

Now, the coordinates of P are

OR
Let the height of parallelogram taking AB as base be h.

Now AB

17. In given figure then prove that

OR

Material downloaded from myCBSEguide.com. 5 / 16


In an equilateral triangle ABC, D is a point on the side BC such that Prove
that

Ans.

Since,

Also
And

OR

Construction: Draw

Material downloaded from myCBSEguide.com. 6 / 16


18. In given figure are two parallel tangents to a circle with centre O
and another tangent AB with point of contact C intersecting at A and at B.
Prove that

Ans. Join OC
In
OP = OC (radii of same circle)
PA = CA (length of two tangents)

AO = AO (Common)
(By SSS congruency criterion)
Hence,
Similarly
Now,

19. Evaluate:

OR

Material downloaded from myCBSEguide.com. 7 / 16


If then evaluate:

Ans.

OR

20. In given figure ABPC is a quadrant of a circle of radius 14 cm and a semicircle is


drawn with BC as diameter. Find the area of the shaded region.

Ans. We know, AC = r

Required area = + ar(semicircle on BC as diameter) –ar(quadrant ABPC

Material downloaded from myCBSEguide.com. 8 / 16


21. Water in a canal, 6 m wide and 1.5 m deep, is flowing with a speed of 10 km/h. How
much area will it irrigate in 30 minutes, if 8 cm of standing water is needed?
OR
A cone of maximum size is carved out from a cube of edge 14 cm. Find the surface area
of the remaining solid after the cone is carved out.
Ans. Let the area that can be irrigated in 30 minute be A
Water flowing in canal in 30 minutes
Volume of water flowing out in 30 minutes
Volume of water required to irrigate the field
Equating (i) and (ii), we get

Or

Surface area of remaining solid where r and l are the radius and slant
height of the cone.

22. Find the mode of the following distribution of marks obtained by the students in an
examination:

Marks obtained 0-20 20-40 40-60 60-80 80-100


Number of students 15 18 21 29 17

Material downloaded from myCBSEguide.com. 9 / 16


Given the mean of the above distribution is 53, using empirical relationship estimate
the value of its median.

Ans.

So, the mode marks is 68.


Empirical relationship between the three measures of central tendencies is:

Section D
Question numbers 23 to 30 carry 4 marks each.

23. A train travelling at a uniform speed for 360 km would have taken 48 minutes less to
travel the same distance if its speed were 5 km/hour more. Find the original speed of
the train.
OR
Check whether the equation has real roots and if it has, find them by
the method of completing the square. Also verify that roots obtained satisfy the given
equation.
Ans. Let original speed of the train be x km/h.
Time taken at original speed
Time taken at increased speed
Now,

OR
Discriminant
So, the given equation has two distinct real roots

Multiplying both sides by 5.

Material downloaded from myCBSEguide.com. 10 / 16


Verification:

Similarly,

24. An AP consists of 37 terms. The sum of the three middle most terms is 225 and the
sum of the last three terms is 429. Find the AP.
Ans. Let the three middle most terms of the AP be a – d, a, a + d.
We have, (a – d) + a + (a + d) = 225

Now, the AP is
a – 18d,…,a – 2d, a – d, a, a + d, a + 2d,…, a + 18d
Sum of last three terms:

Now, first term


The AP is 3, 7, 11, …, 147.
25. Show that in a right triangle, the square of the hypotenuse is equal to the sum of the
squares of the other two sides.
OR
Prove that the ratio of the areas of two similar triangles is equal to the ratio of the
squares of their corresponding sides.
Ans. Given: A right triangle ABC right angled at B.
To prove:
Construction: Draw
Proof: In

Material downloaded from myCBSEguide.com. 11 / 16


Now, (corresponding sides are proportional)

Similarly

Adding (1) and (2)

OR
Given:

To prove:

Construction: Draw

In


Therefore,

But

Hence,

Material downloaded from myCBSEguide.com. 12 / 16


26. Draw a triangle ABC with side Then, construct
a triangle whose sides are times the corresponding sides of
Ans. Draw in which and hence
Construction of similar triangle as shown below:

27. Prove that


Ans.

28. The angles of depression of the top and bottom of a building 50 metres high as
observed from the top of a tower are respectively. Find the height of the
tower and also the horizontal distance between the building and the tower.

Material downloaded from myCBSEguide.com. 13 / 16


Ans.

Now,

Now,
Height of tower = TR = 75 m
Distance between building and tower

29. Two dairy owners A and B sell flavoured milk filled to capacity in mugs of negligible
thickness, which are cylindrical in shape with a raised hemispherical bottom. The mugs
are 14 cm high and have diameter of 7 cm as shown in given figure. Both A and B sell
flavoured milk at the rate of per litre. The dairy owner A uses the formula
to find the volume of milk in the mug and charges for it. The dairy owner B
is of the view that the price of actual quantity of milk should be charged. What
according to him should be the price of one mug of milk? Which value is exhibited by

the dairy owner B?

Ans. Capacity of mug (actual quantity of milk)

Amount dairy owner B should charge for one mug of milk


Value exhibited by dairy owner B: honesty (or any similar value)

Material downloaded from myCBSEguide.com. 14 / 16


30. The following distribution shows the daily pocket allowance of children of a locality.
The mean pocket allowance is Find the missing frequency k.

Daily pocket allowance (in Rs.) 11-13 13-15 15-17 17-19 19-21 21-23 23-25
Number of children 3 6 9 13 k 5 4

OR
The following frequency distribution shows the distance (in metres) thrown by 68
students in a Javelin throw competition.

Distance (in m) 0-10 10-20 20-30 30-40 40-50 50-60 60-70


Number of students 4 5 13 20 14 8 4

Draw a less than type Ogive for the given data and find the median distance thrown
using this curve.
Ans.

Daily pocket allowance Number of children Mid-point


(in Rs.)
11 - 13 3 12 -3 -9
13 – 15 6 14 -2 -12
15 - 17 9 16 -1 -9
17 – 19 13 18 0 0
19 – 21 K 20 1 k
21 – 23 5 22 2 10
23 - 25 4 24 3 12

OR

Less than Number of Students


10 4

Material downloaded from myCBSEguide.com. 15 / 16


20 9
30 22
40 42
50 56
60 64
70 68

Median distance is value of x that corresponds to Cumulative frequency


Therefore, Median distance = 36 m

Material downloaded from myCBSEguide.com. 16 / 16


SAMPLE QUESTION PAPER 01
Class-X (2017–18)
Mathematics

Time allowed: 3 Hours Max. Marks: 80


General Instructions:
(i) All questions are compulsory.
(ii) The question paper consists of 30 questions divided into four sections A, B, C and D.
(iii) Section A contains 6 questions of 1 mark each. Section B contains 6 questions of 2 marks
each. Section C contains 10 questions of 3 marks each. Section D contains 8 questions of 4
marks each.
(iv) There is no overall choice. However, an internal choice has been provided in four
questions of 3 marks each and three questions of 4 marks each. You have to attempt only one
of the alternatives in all such questions.
(v) Use of calculators is not permitted.

Section-A

1. Can two numbers have 18 as their HCF and 380 as their LCM? Give reason.

2. Find the root of the equation .

3. Determine whether 50cm, 80cm, 100cm can be the sides of a right triangle or not.

4. The length of the shadow of a man is equal to the height of man. The angle of elevation is
________.

5. If the perimeter and area of a circle are numerically equal, then find the radius of the
circle.

6. If three coins are tossed simultaneously, then find the probability of getting at least two
heads.

Section-B

7. Is 7× 6 × 5 × 4 × 3 × 2 × 1+ 5 a composite number? Justify your answer.

Material downloaded from myCBSEguide.com. 1 / 17


8. The 11th term of an A.P. exceeds its 4th term by 14. Find the common difference.

9. Find the relation between ‘x’ and ‘y’, if the points , (1,2) and (7,0) are collinear.

10. Two tangents making an angle of 120° with each other are drawn to a circle of radius 6
cm, find the length of each tangent.

11. Prove that

12. A cone of height 20 cm and radius of base 5 cm is made up of modelling clay. A child
reshapes it in the form of a sphere. Find the diameter of the sphere.

Section-C

13. Use Euclid’s division lemma to show that cube of any positive integer is either of the form
9q, 9q+1 or 9q + 8 for some integer ‘q’.

14. Obtain all other zeroes of x4 + 5x3 - 2x2 - 40x -48 if two of its zeroes are

15. Solve for ‘x’: , x ≠ 0.

16. How many terms of the series 54, 51, 48,…….be taken so that their sum is 513? Explain the
double answer.

Or

In an AP pth, qth and rth terms are respectively a, b and c. Prove that
p(b - c) + q(c - a) + r(a - b) = 0

17. The point A(3, y) is equidistant from the points P(6,5) and Q(0, -3). Find the value of y.

Or

If A (4, 6), B (3, -2) and C (5, 2) are the vertices of Δ ABC, then verify the fact that a median of a
triangle ABC divides it into two triangles of equal areas.

18. If prove that

19. An observer 1.5m tall is 28.5m away from a chimney. The angle of elevation of the top of

Material downloaded from myCBSEguide.com. 2 / 17


the chimney from her eyes is 45°. What is the height of the chimney?

Or

From the top of a 7m high building, the angle of elevation of the top of a cable tower is

60o and the angle of depression of the foot of the tower is 30o. Find the height of the tower.

20. A boy is cycling such that the wheels of the cycle are making 140 revolutions per minute.
If the diameter of the wheel is 60 cm, calculate the speed in Km per hour in which the boy is
cycling.

21. The following table shows the gain in weight by 50 children in a year. Calculate modal
gain in weight.

Gain in weight (in kg) 1 - 3 3 - 5 5 - 7 7 - 9 9- 11 11-13

No. of children 4 6 10 18 7 5

Or

Compute the Median for the given data

Class –interval 100-110 110-120 120-130 130-140 140-150 150-160

Frequency 6 35 48 72 100 4

22. What is the probability that a leap year, selected at random will contain 53 Thursdays?

Section-D

23. Solve graphically the following equations 2x + 3y = 9; x – 2y =1. Shade the region bounded
by the two lines and the x axis.

Or

Check graphically whether the pair of equations x + y = 8 and x – 2y = 2 is consistent. If so,


solve them graphically. Also find the coordinates of the points where the two lines meet the
y-axis.

Material downloaded from myCBSEguide.com. 3 / 17


24. A thief away from a Police Station with a uniform speed 100m/minutes. After one minute
a Policeman runs behind the thief to catch him. He goes at a speed of 100m/minute in first
minute and increases the speed 10m/minute on each succeeding minute. After how many
minutes the Policeman catches the thief.
Now answer these questions:
(i) Which mathematical concept is being used to solve the above problem?
(ii) Which trait of personality of the policeman is showed?

25. In the adjoining figure, PQR, is a right triangle, right angled at Q. X and Y are the points

on PQ and QR such that PX : XQ = 1 : 2 and QY : YR = 2 : 1. Prove that 9(PY2 + XR2) = 13 PR2

Or

A quadrilateral ABCD is drawn to circumscribe a circle (fig-2). Prove that, AB + CD = AD + BC.

26. Prove that the lengths of two tangents drawn from an external point to a circle are equal.

27. Construct an isosceles triangle whose base is 7cm and altitude 5 cm and then construct
another triangle whose sides are times the corresponding sides of the isosceles triangle.

28. Suppose a person is standing on a tower of height m and observing a car


coming towards the tower. He observed that angle of depression changes from 30° to 45°, in
3 seconds. Find the speed of the car.

29. A container opens at the top and made up of metal sheet is in the form of a frustum of a

Material downloaded from myCBSEguide.com. 4 / 17


cone of height 16cm with diameters of its lower and upper ends as 16cm and 40cm
respectively. Find the cost of metal sheet used to make the container, if it costs Rs.10 per

100cm2. ( Use =3.14)

30. The mean of the following frequency distribution is 47. Find the value of ‘p’.

Classes 0 - 20 20 - 40 40 - 60 60 - 80 80 - 100

Frequency 8 15 20 p 5

Or

Compute the mode for the following frequency distribution.

Size of items: 0-4 4-8 8-12 12-16 16-20 20-40 24-28 28-32 32-36 36-40

Frequency: 5 7 9 17 12 10 6 3 1 0

Material downloaded from myCBSEguide.com. 5 / 17


CBSE SAMPLE PAPER 01
CLASS X MATHEMATICS
Marking Scheme

1. No. Because HCF is always a factor of LCM but here 18 is not a factor of 380.

2. 16x2 -27x-10=0
(16x + 5) (x - 2)=0

x= , x = 2

3.

Clearly, the sum of the squares of the lengths of two sides is not equal to the square of the
length of the third side. Hence, given sides do not make a right triangle because it does not
satisfy the property of Pythagoras theorem.

4. 45°

5. Perimetre of the Circle = Area of the Circle

6. Number of possible outcomes = 8 ( HHH, HHT, HTH,HTT, THH, THT, TTH, TTT)
Number of favorable outcomes ( 2 head) = 4

So probability =

7. Yes, 5040 + 5 = 5045 It has more than two factors

8. Let the first term of AP is a and d is common difference then According to Question
a11 – a4 =14 ;

=> a + 10d – (a + 3d) = 14


=> a + 10d - a - 3d = 14
=> 7d = 14
=> d=2

9. [ ]

Material downloaded from myCBSEguide.com. 6 / 17


x = 7 – 3y

10.

PT=PS ( length of tangents)


60°

ΔOTP tan 60°=

PT= cm

11. LHS =

= RHS

12. Volume of cone = Volume of sphere

r = 5 cm

Material downloaded from myCBSEguide.com. 7 / 17


13. a = bq + r ; b =3 ; r = 0, 1, 2

a3 =(3m)3

= 9 (3m3)

= 9 q

a3 =(3m+1)3

= 27m3 + 27 m2 + 9m +1

= 9 q + 1

a3 =(3m+2)3

= 27m3 + 54 m2 + 18 m +8

= 9 q + 8

14. Two zeroes are

Therefore

15.

Material downloaded from myCBSEguide.com. 8 / 17


=> 2x2 +2ax +bx +ab = 0

=> 2x( x+a ) + b ( x+a) = 0

=> (x+a) (2x+b) = 0

=> x = -a ,

16. a = 54 , d= -3 , sn = 513

Sn =

513 =

=>
n = 18 , 19

Since d is negative. we get double answer because sum of 18 terms and 19 terms is zero, as
few terms are positive and few are negative.

Or

A + (p - 1)D = a..(i)

A + (q-1) D=b…. (ii)

A + (r-1) D=c… …. (iii)

(ii) - (iii)

b - c = (q-1) D –(r-1)D

Similarly,

Material downloaded from myCBSEguide.com. 9 / 17


Adding (iv), (v) and (vi)

17. PA =QA 17. The point A(3, y) is equidistant from the points P(6,5) and Q(0, -3). Find the value of y.

18.

19.

Material downloaded from myCBSEguide.com. 10 / 17


Given the height of the observer be DE = 1.5 m
That is AB = 1.5 m
Let BC = h is height of the chimney
Hence AC = (h – 1.5) m

Given distance between the observer and the chimney is

AD = BE = 28.5 m
In right triangle DCA, θ = 45°
tan 45°=

∴ h = 28.5 +1.5 = 30 m
Thus the height of the chimney is 30 m.

20. Circumference of wheel = πd = 60 π cm

Distance covered in 1 revolution = km

Distance covered in 140 revolution = km

(Distance covered in 1 min)

Distance covered in 1 hr =
Speed of cycle = 15.84 km / hr

21. Modal class = 7 - 9

Mode-

Material downloaded from myCBSEguide.com. 11 / 17


=

22. There are 366 days in a leap year that contain 52 weeks and 2 more days. So, 52
Thursdays and 2 days.
These 2 days can be:
(Mon, Tue}, {Tue, Wed}, {Wed, Thu}, {Thu, Fri}, {Fri, Sat}, {Sat, Sun} and {Sun, Mon} (7 cases).
In order to have 53 Thursdays we should have either {Thu, Fri} or {Wed, Thu} case.
No. of sample spaces = 7.
No. of event that gives 53 Thursdays in a leap Year = 2.
Required Probability =

23.

Table for Eqn.1 and its line


Table for Eqn.2 and its line
Solution x =3 and y =1
Shaded area is required solution.

Or

x 0 4 8

Material downloaded from myCBSEguide.com. 12 / 17


y = 8 –x 8 4 0

Three solutions for equation (i)are given in the table :

Three solutions for equation (ii) are given in the table :

x 0 2 8

–1 0 3

Drawing Line AC

Drawing Line PR

Plotting points A(0, 8), B(4, 4) and C(8, 0) on graph paper the straight line AC is obtained as
graph of the equation
(i) Plotting points P(0, –1), Q(2, 0) and R(8, 3) on graph paper the straight line PR is obtained
as graph of the equation
(ii). From the graph, it is clear that a point M(6, 2) common to both the lines AC and PR.

So the pair of equations is consistent and the solutions of the equations are x = 6 and y = 2.
From the graph it is seen that the coordinates of the points where the lines AC and PR meets
the y-axis are (0, 8) and (0, –1) respectively.

24. Time taken by Thief before being caught = n+1

Material downloaded from myCBSEguide.com. 13 / 17


Distance travelled by Thief = 100 ( n+ 1)
100 ( n+1) =

n = 5 minutes
i) Arithmetic Progression
ii) Responsibility of their work ( duty) and honesty

25.

Or

Since, the lengths of tangents drawn from an external point to a circle are equal.

AP = AS … (i) BP = BQ … (ii)

Material downloaded from myCBSEguide.com. 14 / 17


CQ = CR … (iii) DR = DS … (iv)

Now, AB + CD

= AP + PB + CR + RD

= AS + BQ + CQ + DS

= (AS + DS) + (BQ + CQ)

= AD + BC

Hence proved.

26. Construction: Draw a circle with centre O. From a point P outside the circle, draw two
tangents P and R.

To Prove: PQ = PR

Proof: In Δ POQ and Δ POR

OQ = OR (radii)

PO = PO (common side)

∠PQO = ∠PRO (Right angle)

Δ POQ Δ POR (By RHS Congruency rule)

Hence proved

Material downloaded from myCBSEguide.com. 15 / 17


27.

28.

Δ PQB tan 45° = , x= 15( ------------ (1)

Δ PQA tan 30° = x + y = 15 ( --------- (2)

From (1) and (2) y = 30 m

Since the car moving from A to B in 3 seconds

Speed = = 10 m / sec

29. R = 20 cm , r = 8 cm , h= 16 cm

l = = 20 cm

Material downloaded from myCBSEguide.com. 16 / 17


Total surface area = CSA of frustum + area of base
= πl(R+r) +
= 1959.36

Rate of metal sheet used = Rs.10 per 100 cm2


Cost of metal sheet used = 1959.36 × = Rs.195.94 (Approximately)

30.

CI fi xi fix i

0 - 20 8 10 80

20 - 40 15 30 450

40 - 60 20 50 1000

60 - 80 p 70 70p

80 - 100 5 90 450

48+p 1980+70p

Material downloaded from myCBSEguide.com. 17 / 17


SAMPLE QUESTION PAPER 02
Class-X (2017–18)
Mathematics

Time allowed: 3 Hours Max. Marks: 80


General Instructions:
(i) All questions are compulsory.
(ii) The question paper consists of 30 questions divided into four sections A, B, C and D.
(iii) Section A contains 6 questions of 1 mark each. Section B contains 6 questions of 2 marks
each. Section C contains 10 questions of 3 marks each. Section D contains 8 questions of 4
marks each.
(iv) There is no overall choice. However, an internal choice has been provided in four
questions of 3 marks each and three questions of 4 marks each. You have to attempt only one
of the alternatives in all such questions.
(v) Use of calculators is not permitted.

SECTION - A

1. The sum and the product of zeroes of a quadratic polynomial p(x) are 7 and 10
respectively. Then find p(x).

2. Which term of the A.P 92, 88, 84, 80, . . . . is 0?

3. Find the ratio in which the Y axis divides the line segment joining the points (5, -6) and (-1,
-4).

4. If PT is a tangent drawn from a point P to a circle touching it at T, and O is the centre of the
circle, then OPT + POT is ____________

5. If two towers of heights h1 and h2 subtend angles of 60° and 30° respectively, at the mid-

point of the line joining their feet, then h1 : h2 is ___________

6. In rolling a dice, the probability of getting a number less than 4 is ____________

SECTION - B

Material downloaded from myCBSEguide.com. 1 / 18


7. Solve for ‘x’ by completing the square method:

8. Find the 7th term from the end of the A.P. 7, 10, 13, . . . 184.

9. Find ‘a’ so that (3, a) lies on the line represented by 2x - 3y - 5 = 0. Also, find the coordinates
of the point where the line cuts the x axis.

10. ABC is a right triangle right-angled at B. Let D and E be any points on AB and BC

respectively. Prove that AE2 + CD2 = AC2 + DE2.

11. Two concentric circles have radii 5 cm and 3 cm. Find the length of the chord of the
larger circle which touches the smaller circle.

12. A cylinder and a cone have base radii 5 cm and 3 cm, respectively, and their respective
heights are 4 cm and 8 cm. Find the ratio of their volumes.

SECTION - C

13. A part of monthly expenses of a family is constant and the remaining varies with the
price of rice. When the cost of rice is Rs.250 per quintal, the monthly expenditure of the
family is Rs.1000 and when the cost of rice is Rs.240 per quintal the monthly expenditure is
Rs.980. Find the monthly expenditure of the family when the cost of rice is Rs.300 per
quintal.

14. Prove that is irrational.

15. Use Euclid’s division algorithm to find the HCF of 4052 and 12576.

16. The Sum Sn of first ‘n’ even natural numbers is given by the relation Sn= n(n+1). Find n, if

the sum is 420.

Or

A man arranges to pay a debt of Rs.3600 in 40 monthly installments which are in a AP. When
30 installments are paid he dies leaving one third of the debt unpaid. Find the value of the
first installment.

17. Find the coordinates of the points of trisection of the line segment joining the points A( 1,

Material downloaded from myCBSEguide.com. 2 / 18


-2) and B(-3, 4).

Or

If the points (p, q) (M, n) and (p – m, q –n) are collinear, show that pn = qm.

18. In an equilateral triangle ABC, if AD BC, then prove that 3AB2= 4AD2.

19. In the adjoining figure, two tangents PQ and PR are drawn to a circle with Centre O from
an external point P. Prove that

Or

A sector of circle of radius 12 cm has the angle 120⁰ . It is rolled up so that the two bounding
radii are formed together to form a cone. Find the volume of the cone.

20. Find the value of

21. Two different dice are thrown together. Write all the possible outcomes. Find the
probability that the product of the numbers appearing on the top of the dice is less than 9.

Or

Anil bags contain 5 red marbles, 8 white marbles and 4 green marbles. One marble is taken
out of the box at random. What is the probability that the marble taken out will be
(i) Red
(ii) White
(iii) Not green

22. The king, queen and jack of clubs are removed from a deck of 52 playing cards. The
remaining cards are well shuffled and one card is drawn at random from it. Find the
probability of getting the selected card as (i)a heart (ii). a king (iii). a club

Material downloaded from myCBSEguide.com. 3 / 18


SECTION - D

23. A two-digit number is such that the product of its digits is 12. When 36 is added to this
number, the digits interchange their places. Find the number.

Or

Solve for x, y:

24. A contract on construction job specifies a penalty for delay of completion beyond a
certain date as follows: Rs. 200 for the first day, Rs. 250 for the second day, Rs.300 for the
third day, etc., the penalty for each succeeding day being Rs.50 more that for preceding day.

(i) How much money the contractor has to pay as penalty, if he has delayed the work by 30
days?
(ii) Which moral value one can learn from above problem?

25. Draw a pair of tangents to a circle of radius 4 cm, which are inclined to each other at an
angle of 60°.

Or

Construct a tangent to a circle of radius 2 cm from a point on the concentric circle of radius
2.6 cm and measure its length. Also verify the measurements by actual caclulations.(length of
tangent = 2.1 cm)

26. Prove that

27. In the adjoining figure four equal circles are described at the four corners of a square so

that each touches two of the others. The shaded area enclosed between the circles is cm2.
Find the radius of each circle.

Material downloaded from myCBSEguide.com. 4 / 18


28. The angle of elevation of an aeroplane from a point A on the ground is 600. After a flight

of 30 seconds, the angle of elevation changes to 300. If the plane is flying at a constant height
of m, find the speed in km/hr of the plane.

29. A right triangle having sides 15cm and 20cm is made to revolve about its hypotenuse.
Find the volume and Surface Area of the double cone so formed. (Use p =3.14)

30. During the medical check-up of 35 students of a class, their weights were recorded as
follows

Weight Less Less Less Less Less Less Less Less


(in kg) than 38 than 40 than 42 than 44 than 46 than 48 than 50 than 52

No. of
0 3 5 9 14 28 32 35
Students

Draw a less than type ogive for the given data. Hence, obtain the median weight from the
graph and verify the result by using the formula.

Or

Compute the Mean and Median for the given data.

Class –interval 100-110 110-120 120-130 130-140 140-150 150-160

Frequency 6 35 48 72 100 4

Material downloaded from myCBSEguide.com. 5 / 18


CBSE SAMPLE PAPER 02
CLASS X - Mathematics
Marking Scheme

1. x2- 7x+10

2. 24th term is 0.

3. 5 : 1

4. 90°

5. 3 : 1

6.

7.

8. d = -3 ; a = 184

a7=a + 6d

= 184 + 6 (-3)

= 184 – 18

= 166

9. Since (3, a) lies on 2x – 3y -5 = 0,

2(3) – 3a – 5 =0

a = 1/3

Material downloaded from myCBSEguide.com. 6 / 18


Let the co-ordinate of the point which cuts the x axis be (x, 0).

2x – 3(0)-5 = 0.

x = 5/2

(5/2, 0) is the point of intersection with x – axis

10.

Since ABE is right triangle, right-angled at B

AE2 = AB2 + BE2 ...(i)

Again, DBC is right triangle right- angled at B

CD2 = BD2 + BC2 ...(ii)


Adding (i) and (ii) we get

AE2 + CD2 = (AB2 + BE2 )+( BD2 + BC2 )

AE2 + CD2 = (AB2 + BC2 )+( BE2 + BD2 )

Using Pythagoras theorem in ABC and DBE we have

AE2 + CD2 = AC2+DE2

11.

Material downloaded from myCBSEguide.com. 7 / 18


Here PQ is tangent to smaller circle.

OR PQ

Now in PRO,

PR2+OR2=OP2

PR2+32=52

PR=

PR = QR (Perpendicular from the centre of the circle bisects the chord)

Length of the chord of the larger circle = PQ = 2 PR = 2 × 4 = 8 cm.

12. Volume of cylinder : Volume of cone

= 25 : 6

13. Let the constant expenditure be Rs.x

Let the consumption of rice by the family be y quintals.

x + 250 y = 1000------------------(1)

x + 240 y = 980-------------------(2)

Material downloaded from myCBSEguide.com. 8 / 18


Solving (1) and (2)

x = 500; y = 2

Total expenditure when the cost of rice is Rs.300 = x +300y

= Rs.1100

14. Suppose is rational

But irrational while is rational and an irrational number can never be equal to a
rational number.

Thus our assumption is wrong and hence is irrational

15. Since 12576 > 4052, we apply the division lemma to 12576 and 4052, to get

12576 = 4052 × 3 + 420

4052 = 420 × 9 + 272

420 = 272 × 1 + 148

272 = 148 × 1 + 124

148 = 124 × 1 + 24

124 = 24 × 5 + 4

24 = 4 × 6 + 0

The remainder has now become zero, so our procedure stops.

HCF of 12576 and 4052 is 4.

Material downloaded from myCBSEguide.com. 9 / 18


16. n (n+1)=420

n2 + n – 420 = 0

n = -21 , 20

n cannot be negative ∴ n=20

17.

Let P(x1,y1) , Q(x2, y2) divide AB into 3 equal parts.

P divided AB in the ratio of 1 : 2

P(x1, y1) =

Since Q is the midpoint of PB,

Q(x2, y2) = =

18.

Given: ABC is equilateral and AD BC.

To Prove: 3AB2 = 4 AD2

Material downloaded from myCBSEguide.com. 10 / 18


Proof:

In ABD, By Pythagoras theorem,

AB2 = AD2 + BD2

BD =

AB2 =

AB2

19. Given: - (i) PQ and PR are the tangents of the circle


(ii) O is the centre of the circle

To prove:
Construction : Join OR and OP

Proof : - Now OR PR (radius from point of contact is to tangent)

Similarly, OQ PQ

In quadrilateral OPQR

(angles in same segment) ….. (1)

Also
[from (1)]
………. (2)

From (1) and (2)

Material downloaded from myCBSEguide.com. 11 / 18


Hence proved

20.

= 1 + 1 – 2

= 0

20. S= { (1,1) , (1,2), (1,3), (1,4), (1,5), (1,6),

( 2,1) , (2,2) , (2,3) , (2,4), (2,5) , (2,6),

(3,1) , (3,2) , (3,3) , (3,4), (3,5) , (3,6),

(4,1) , (4,2) , (4,3) , (4,4), (4,5) , (4,6),

(5,1) , (5,2) , (5,3) , (5,4), (5,5) , (5,6),

(6,1) , (6,2) , (6,3) , (6,4), (6,5) , (6,6)}

n(S) = 36

Favourable events:(1,1) , (1,2) , (1,3) , (1,4), (1,5) , (1,6), ( 2,1) , (2,2) ,(2,3) , (2,4),(3,1) , (3,2) , (4,1)
, (4,2) , (5,1), (6,1)

n(E) = 16

P(E) =

22.
Total number of cards = 52

Number of cards removed = 3

n(s) = 52- 3 = 49

Material downloaded from myCBSEguide.com. 12 / 18


i) P( heart) =

ii) P(a king) =

iii) P( a club ) =

23. Let Two digit number = 10x + y

xy=12 ------- (1)

10x + y + 36 = 10y + x

x – y = - 4 -------- (2)

from 1 and 2 we get x2 + 4x – 12 =0

x= -6 , 2

x cannot be negative , so x= 2

y=12 / 2 = 6

Required number is 26

24. The required sequence is 200, 250, 300, 350, ..........

a = 200, d = 50, n = 30

The contractor has to pay Rs. 27,750.

Value: One should be punctual and show dedication to his work, failing of which may result
loss.

25.

Material downloaded from myCBSEguide.com. 13 / 18


Draw a circle with radius 4 cm.

Draw 2 radius OQ and OR at 120°.

Construct s at Q, R to meet at P.

Join OQ and OR and they are required tangents.

26. LHS =

Material downloaded from myCBSEguide.com. 14 / 18


27.

Let r cm be the radius of each circle


Area of square - Area of 4 sectors =

Radius of each circle is 2 cm. (radius cannot be negative)

6r2 =24

r2 =4

r = ±2

28.

Let P be the position of the flight initially.

And BP be the height of the flight from the ground.

Consider ABP,

Material downloaded from myCBSEguide.com. 15 / 18


Consider ACQ,

BC = AC – AB=10,800 – 3600 = 7200 m

Speed of the flight =

= 864 km/hr

29. OA = 12 cm

Material downloaded from myCBSEguide.com. 16 / 18


30. By locating upper limits on X axis and number of students on Y axis, we get the following
graph

From the graph, we see that median is 46.5.

The frequency distribution table with the given cumulative frequencies becomes:

Class Intervals Frequency Cumulative Frequency

Below 38 0 0

38-40 3 3

40-42 2 5

42-44 4 9

44-46 5 14

Material downloaded from myCBSEguide.com. 17 / 18


46-48 14 28

48-50 4 32

50-52 3 35

Here n/2 = 35/2 = 17.5.

Now 46 – 48 is the class whose cumulative frequency is 28 is greater than n/2 i.e17.5.

So, 46 – 48 is the median class.

From the table f = 14, cf = 14, h =2

Median =

= 46 + 0.5

= 46. 5

Material downloaded from myCBSEguide.com. 18 / 18


SAMPLE QUESTION PAPER 03
Class-X (2017–18)
Mathematics

Time allowed: 3 Hours Max. Marks: 80


General Instructions:
(i) All questions are compulsory.
(ii) The question paper consists of 30 questions divided into four sections A, B, C and D.
(iii) Section A contains 6 questions of 1 mark each. Section B contains 6 questions of 2 marks
each. Section C contains 10 questions of 3 marks each. Section D contains 8 questions of 4
marks each.
(iv) There is no overall choice. However, an internal choice has been provided in four
questions of 3 marks each and three questions of 4 marks each. You have to attempt only one
of the alternatives in all such questions.
(v) Use of calculators is not permitted.

Section A

Q.1 Find the value of so that the point , lies on the line represented by

Q.2 Write the next term of the AP:

Q.3 If the value of .

Q.4 If areas of two similar triangles are in ratio 25: 64, write the ratio of their corresponding
sides.

Q.5 In figure, CP and CQ are tangents to a circle with centre O . ARB is another tangent
touching the circle at R. If CP , BC = 7 cm, then find the length of BR .

Q.6 A bag contains 4 red and 6 black balls. A ball is taken out of the bag at random. Find the
probability of getting a black ball.

Material downloaded from myCBSEguide.com. 1 / 26


Section B

Q.7 Prove that is an irrational number.

Q.8 Solve:

Q.9 Find the value of p so that the quadratic equation has two equal
roots.

Q.10 Evaluate :

Q.11 The angle of elevation of the top of the tower from a point on the ground, which is 30 m
away from the foot of the tower, is 30 . Find the height of the tower.

Q.12 A solid sphere of radius 10.5 cm is melted and recast into smaller solid cones, each of

radius 3.5 cm and height 3 cm. Find the number of cones so formed.

Section C

Q.13 In a flight of 2800 km, an aircraft was slowed down due to bad weather. Its average
speed is reduced by 100 km/h and time increased by 30 minutes. Find the original duration
of the flight.

Q.14 The sum of 4th and 8th term of an AP is 24 and sum of 6th and 10th terms is 44. Find AP.

Q.15 If one diagonal of trapezium divides the other diagonal in the ratio 1: 3. Prove that one
of the parallel sides is three times the other.

Q.16 Find the area of quadrilateral ABCD whose vertices are


.

Or

The points A(4, –2), B(7, 2), C(0, 9) and D(–3, 5) form a parallelogram. Find the length of the
altitude of the parallelogram on the base AB.

Material downloaded from myCBSEguide.com. 2 / 26


Q. 17 An aeroplane, when 3000 m high passes vertically above plane at any instant, when the
angle of elevation of the two aeroplanes from the same point on the ground are and
respectively. Find the vertical distance between aeroplanes.

Or

The angles of depression of the top and bottom of a building 50 metres high as observed from
the top of a tower are 30° and 60°, respectively. Find the height of the tower and also the
horizontal distance between the building and the tower.

Q.18 If all sides of a parallelogram touch a circle, show that the parallelogram is rhombus.

Q.19 Construct a triangle ABC in which BC = 8 cm, B = and C . Construct


another triangle similar to ABC such that its sides are of the corresponding sides of
ABC.

Or

Draw a triangle ABC with side BC = 7 cm, . Then, construct a triangle


whose sides are times the corresponding sides of ABC.

Q.20 Show that the point A (3, 5), B (6, 0), C (1, -3) and D (-2, 2) are the vertices of a square
ABCD.

Q.21 The median of distribution given below is 14.4. Find the values of and , if the sum of
frequency is 20.

Class interval 0-6 6-12 12-18 18-24 24-30

Frequency 4 x 5 y 1

Or

Find the mean, mode and median for the following data:

Classes 10-20 20-30 30-40 40-50 50-60 60-70 70-80

Frequency 4 8 10 12 10 4 2

Material downloaded from myCBSEguide.com. 3 / 26


Q.22. Two dice are thrown simultaneously. What is the probability that
a) 5 will not come up on either of them?
b) 5 will come up on at least one?
c) 5 will come up on at both Dice?

Section D

Q.23 Use Euclid’s Division Algorithm to show that the square of any positive integer is either
of the form or 3m+1 for some integer m.

Or

Show that exactly one of the numbers n, n + 2 or n + 4 is divisible by 3.

Q.24 if the polynomial is divided by the


reminder comes out to be

Q.25 Draw the graphs of the following equations: x + y = 5, x - y = 5


(i) Find the solution of equation from the graph.
(ii) Shade the triangular region formed by the lines and y-axis.

Or

Find the solution of pair of linear equation with the help of graph.2x + y = 6, 4x − 2y = 4

Q.26 In an equilateral triangle ABC, D is a point on side BC such that 4BD = BC. Prove that

Q.27 If prove that

Q.28 In the given figure, three circle each of radius 3.5 cm are drawn in such a way that each
of them touches the other two. Find the area of shaded region enclosed between these three
circles. [Use

Material downloaded from myCBSEguide.com. 4 / 26


Q.29 Water is flowing at the rate of 15 km/hour through a pipe of diameter 14 cm into a
cuboidial pond which is 50 m long and 44 m wide. In what time will the level of water in
pond rise by 21 cm?

Q. 30 Find the mean, mode and median of the following frequency distribution :

Class 0-10 10-20 20-30 30-40 40-50 50-60 60-70

Frequency 4 4 7 10 12 8 5

Or

The following distribution shows the daily pocket allowance of children of a locality. The
mean pocket allowance is Rs18. Find the missing frequency k.

Daily Pocekt
11-13 13-15 15-17 17-19 19-21 21-23 23-25
Allowence

Number of
3 6 9 13 k 5 4
Children

Material downloaded from myCBSEguide.com. 5 / 26


CBSE SAMPLE PAPER 03
CLASS X Mathematics
Solutions

Solution 1: Since point lies on line

Then

Solution 2:

AP is and so on

Common difference

Next term

Solution 3:

Solution 4: We know that,

If two triangle are similar then,

Material downloaded from myCBSEguide.com. 6 / 26


Solution 5: CP = CQ = 11 cm

[Tangents from same external point]

CQ = BC + BQ

ButBQ = BR

11 = 7 + BR [as BC = 7 cm ]

BR = 4 cm

Solution 6: Number of ways t0o select a ball = 10

Number of ways to select a black ball = 6

Probability of getting a black ball =

Solution 7 Let, If possible, is a rational number.

From , we notice

LHS is an irrational number and RHS is rational number, which is not possible.

Hence, our supposition is wrong. Hence,

Is an irrational number.

Solution 8:

Adding (i) and (ii) we get


Material downloaded from myCBSEguide.com. 7 / 26


Subtract (i) from (ii) we get

Solving and we get

Solution 9:

Here,

For equal roots,D=0

But m [ In quadratic equation, q ]

Solution 10:

Solution 11: Let h be the height of the tower

In

Material downloaded from myCBSEguide.com. 8 / 26


Solution 12: Volume of solid sphere

Volume of 1 cone

Number of cones formed

Solution 13: Let original duration of flight be hours.


Distance km
Usual speed
When time is increased, then time taken hrs
And new speed

A.T.Q. ,

Original duration of the flight hours

Solution 14: Let the 4th term

Material downloaded from myCBSEguide.com. 9 / 26


And 8th term

Then their sum

Similarly

On subtracting equation from we get

[From ]

Therefore, AP is

Solution 15:

DE = EB = 1 : 3

Material downloaded from myCBSEguide.com. 10 / 26


In (alt. angles)

(V.O.A.)

Solution 16:

Area of

Similarly, area of

Area of quadrilateral ABCD Area of Area of

Or

Let the height of parallelogram taking AB as base be h.


Now AB =

Area of
=

Material downloaded from myCBSEguide.com. 11 / 26


Now,

Solution 17: one aeroplane flying at point

Another aeroplane flying at point C. Both are in the same line.

In right

In right

From

Since

Vertical distance between two aeroplane = 3000-1732 m

Material downloaded from myCBSEguide.com. 12 / 26


Or

In

As BP = GR then From (i) and (ii)

Now, TR = TP + PR = (25 + 50) m.


Height of tower =TR = 75 m.
Distance between building and tower = GR =

Material downloaded from myCBSEguide.com. 13 / 26


Solution 18:

Sides of parallelogram ABCD, touch circle at P, Q, R, S

AP = AS, BP = BQ, CQ =CR, DR = DS


[Length of the tangents from a point outside the circle is equal ]

Consider, AB + CD AP + PB + CR + DR

AS + BQ +CQ +DS

(AS +SD ) + (BQ +QC)

AD + BC

AB + AB AD + AD
[ Opposite sides of a parallelogram are equal ]

2 AB 2AD

AB AD
AS Adjacent sides of a parallelogram are equal. Hence parallelogram is a rhombus

Solution 19:

Material downloaded from myCBSEguide.com. 14 / 26


Steps of construction:
1. Draw a line segment BC = 8 cm.

2. Then construct B = at B.

3. Then construct C = at C.

4. Line segment from the angles B and C, when produced, meet at A.

5. ABC is the constructed triangle.

6. Draw an acute angle CBX below BC.

7. Take points at BX, such that

8. Join

9. Draw parallel to meeting BC at C’.

10. Draw C’A’ parallel to CA, meeting BA at A’.

Or

Draw ABC in which BC = 7 cm, and hence C = 30°. Construction


of similar triangle A'BC' as shown below:

A’BC’ is the required triangle similar to ABC where each side is of the side of ABC

Material downloaded from myCBSEguide.com. 15 / 26


Solution 20:

AB

BC

Solution 21:

Class interval Frequency cf

0-6 4 4

6-12 x 4+x

12-18 5 9+x

18-24 y 9+x+y

24-30 1 10+x+y

20

Median

Median class

Median

Material downloaded from myCBSEguide.com. 16 / 26


14.4

Or

We have

Classes Mid value (xi) fi fiu i c.f

10-20 15 4 -3 -12 4

20-30 25 8 -2 -16 12

30-40 35 10 -1 -10 22

40-50 45 12 0 0 34

50-60 55 10 1 10 44

60-70 65 4 2 8 48

70-80 75 2 3 6 50

Let assumed mean a = 45, Here h = 10

We know that mean

Mean

Since maximum frequency = 12

Modal class = 40-50

Material downloaded from myCBSEguide.com. 17 / 26


Here,

Now Mode

Mode = 45

Now

Median class is 40-50

Now median

Here

Median =

Thus, Median = 42.5

Solution 22: Elementary Events are

Material downloaded from myCBSEguide.com. 18 / 26


Total number of elementary events =36

a) Let A=5 will not come up on either of them.

Total number of elementary events favorable to A=25

b) Let B=5 will come up at least one die.

Total number of elementary events favorable to B=11

c) Let C=5 will come up at both dice.

Total number of elementary events favorable to C=1

Solution 23: Let n be any positive integer.

When

Material downloaded from myCBSEguide.com. 19 / 26


When N

(Where )

When

(Where

Square of any positive integer is either of the form

Solution 24: P ( )

( )

Material downloaded from myCBSEguide.com. 20 / 26


Equating the coefficients, we get

Also

Using

From and we get

Solution 25:

5 0

0 5

5 0

0 5

(i) Both lines intersect at point (5,0) on x- axis

Material downloaded from myCBSEguide.com. 21 / 26


(ii) Required portion is shaded in the graph.

Or

2x + y = 6, 4x − 2y = 4

For equation 2x + y – 6 = 0, we have following points which lie on the line

x 0 3

y 6 0

For equation 4x – 2y – 4 = 0, we have following points which lie on the line

x 0 1

y -2 0

We can clearly see that lines are intersecting at (2, 2) which is the solution.

Hence x = 2 and y = 2 and lines are consistent.

Solution 26: In equilateral

4BD=BC

Material downloaded from myCBSEguide.com. 22 / 26


Construction: Draw

In right AED,

In right

[using (i)]

Solution 27:

Now, LHS

Solution 28:

Material downloaded from myCBSEguide.com. 23 / 26


ABC is an equilateral triangle each of whose side is of length = 3.5 + 3.5 = 7 cm

ar( ABC) =

=
Area of 3 sectors

Area of shaded region

Solution 29: let the level of water rises in the tank in hours

Length of the water flow in hours = 15000 metres

Diameter of the pipe = 14 cm

Radius

Volume of water

Volume of water flow in hours in the pond

Material downloaded from myCBSEguide.com. 24 / 26


Volume of cuboidal pond with water level of height 21 cm

ATQ,

Hence, the level of water in the pond will rise by 21 cm in 2 hours.

Solution 30: Table for mean, mode and median :

C.I

0-10 5 4 -3 -12 4

10-20 15 4 -2 -8 8

20-30 25 7 -1 -7 15

30-40 35 10 0 0 25

40-50 45 12 1 12 37

50-60 55 8 2 16 45

60-70 65 5 3 15 50

Mean

For Median :

Median Class :

Median

Material downloaded from myCBSEguide.com. 25 / 26


For Mode: Maximum frequency

Modal Class is

Mode

Or

Daily Pocket Number of Mid-point


f iu i
Allowance Children(fi) (xi)

11-13 3 12 -3 -9

13-15 6 14 -2 -12

15-17 9 16 -1 -9

17-19 13 18 0 0

19-21 k 20 1 k

21-23 5 22 2 10

23-25 4 24 3 12

Material downloaded from myCBSEguide.com. 26 / 26


SAMPLE QUESTION PAPER 04
Class-X (2017–18)
Mathematics

Time allowed: 3 Hours Max. Marks: 80


General Instructions:
(i) All questions are compulsory.
(ii) The question paper consists of 30 questions divided into four sections A, B, C and D.
(iii) Section A contains 6 questions of 1 mark each. Section B contains 6 questions of 2 marks
each. Section C contains 10 questions of 3 marks each. Section D contains 8 questions of 4
marks each.
(iv) There is no overall choice. However, an internal choice has been provided in four
questions of 3 marks each and three questions of 4 marks each. You have to attempt only one
of the alternatives in all such questions.
(v) Use of calculators is not permitted.

SECTION - A

1. Find the zeroes of the quadratic polynomial .

2. Write the sum of the first fifteen natural numbers.

3. Find the coordinate of the mid-point of the line segment joining the points whose

coordinates are and .

4. In fig-1, centre of the circle is O. From outside point P, two tangents PA and PB are drawn

to touch the circle. Given ÐAPB = 60o. Find ÐAOB.

5. A ladder of length 4 m. makes an angle of 45o with the ground when placed against an

Material downloaded from myCBSEguide.com. 1 / 21


electric post. Determine the distance between the feet of the ladder and the electric post.

6. Captain of Indian cricket team tosses two different coins, one of ₹1 and other of ₹2
simultaneously. What is the probability that he gets at least one head?

SECTION - B

7. Find the discriminant of the equation and hence find the nature its
roots.

8. If the 10th term of an A.P. is 47 and its 1st term is 2, find the sum of its first 15 terms.

9. Find a relation between x and y such that the point (x, y) is equidistant from the points (7,
1) and (3, 5).

10. In fig-2, and . Prove that, is isosceles.

11. Two concentric circles are of radius 5 cm and 3 cm. Find the length of the chord of the
larger circle which touches the smaller circle.

12. The height and base diameter of a solid cylinder are 210 m and 24 cm respectively. Find
the volume of the cylinder.

SECTION - C

13. The taxi charges in a city consist of a fixed charge together with the charge for the
distance covered. For a distance of 10 km, the charge paid is ₹ 200 and for a journey of 15
km, the charge paid is ₹275. What are the fixed charges and the charge per km? How much a
person has to pay for travelling a distance of 25 km?

Material downloaded from myCBSEguide.com. 2 / 21


14. Show that is irrational.

15. Show that any positive odd integer is of the form 4q + 1 or 4q + 3, where q is some integer.

16. The 4th term of an A.P. is equal to three times the 1st term & the 7th term exceeds twice

the 3rd term by 1. Find the 1st term and common difference.

Or

The first and the last terms of an AP are 17 and 350 respectively. If the common difference is
9, how many terms are there and what is their sum?

17. If A(-5, 7), B(- 4, -5), C(-1, -6) and D(4, 5) are the vertices of a quadrilateral, find the area.

Or

Find the coordinates of the points which divide the line-segment joining the points (-4, 0) and
(0, 6) in four equal parts.

18. Prove that the sum of the squares of the sides of a rhombus is equal to sum of the squares
on its diagonals

19. Prove that, the lengths of tangents drawn from an external point to a circle are equal.

Or

In fig-3, PQ and RS are two parallel tangents to a circle with centre O and another tangent EF
with point of contact C intersecting PQ at E and RS at F. Prove that ∠EOF = 90°.

20. Evaluate:

Material downloaded from myCBSEguide.com. 3 / 21


21. A box contains 90 discs which are numbered from 1 to 90. If one disc is drawn at random
from the box, find the probability that it bears a perfect square number.

Or

Two horses are considered for a race. The probability of selection of the first horse is 1/4 and
that of second is 1/3. What is the probability that :
(a) both of them will be selected.
(b) only one of them will be selected.
(c) none of them will be selected.

22. A box contains 5 red marbles, 8 white marbles and 4 green marbles. One marble is taken
out of the box at random. What is the probability that the marble taken out will be
i) white ii) not green?

SECTION - D

23. Two water taps together can fill a tank in hours. The tap of larger diameter takes 10
hrs less than the smaller one to fill the tank separately. Find the time in which each tap can
separately fill the tank.

Or

A train, travelling at a uniform speed for 360 km, would have taken 48 minutes less to travel
the same distance if its speed were 5 km/hr more. Find the original speed of the train.

24. Find the sum of all three-digit numbers which leaves the remainder 3 when divided by 5.

25. Construct a triangle similar to triangle ABC with its side equal to of the corresponding
sides of the triangle ABC.

Or

Construct an isosceles triangle whose base is 7cm and altitude 5 cm and then construct
another triangle whose sides are times the corresponding sides of the isosceles triangle.

26. Prove that: .

Material downloaded from myCBSEguide.com. 4 / 21


27. In fig-3, of ABC is a right angle and AB = 3 units, AC = 4 units. Semicircles are
drawn on AB, AC and BC as diameters. Also a circle circumscribing the ABC is drawn. Find
the area of the shaded region.

28. Two ships are sailing in the sea on either side of a light-house. The angles of depression of

two ships as observed from the top of the light-house are 60o and 45o respectively. If the

distance between the ships is meters, find the height of the light-house.

29. A person connects a pipe of internal diameter 20m from a canal into a empty cylindrical
tank in his field which is 10 m in diameter and 2 m deep. If the rate of flow of water through
the pipe is 6km/hr, then after how much time the flow of water should be stopped to avoid
over flow of water from the tank ? What value we have from the problem ?

30. The median of the following frequency distribution is 35. Find the value of x.

Class Interval 0-10 10-20 20-30 30-40 40-50 50-60 60-70

Frequency 2 3 5 6 x 3 2

Or

Compute the mode for the following frequency distribution.

Size of items: 0-4 4-8 8-12 12-16 16-20 20-40 24-28 28-32 32-36 36-40

Frequency: 5 7 9 17 12 10 6 3 1 0

Material downloaded from myCBSEguide.com. 5 / 21


CBSE SAMPLE PAPER 04
CLASS X Mathematics
Solution

SECTION- A

1. We have

=(x+5)(x+2)

So, the zeroes of are - 5 and - 2 .

2. The sum of the first fifteen natural numbers =

3. Coordinates of the mid-point are

4.

5.

6. Possible outcomes are (H, H), (H, T), (T, H), (T, T) which are equally likely.
Outcomes favorable to the event are (H, H), (H, T), (T, H).Hence the required Probability =

SECTION- B

7. Here, .

Discriminant

Material downloaded from myCBSEguide.com. 6 / 21


Hence the given quadratic equation has two equal real roots.

8. Let the common difference of the A.P be d.

Given, the first term (a) = 2 and the 10th term ( ) = 47

As, nth term ,

So,

i.e., 9d = 47 - 2

i.e.,

As, the sum of the first n-terms of an AP,

Therefore, the sum of the first 15-terms of the AP

9. Since the point P(x , y) is equidistant from the points A(7, 1) and B(3, 5)

So, PA = PB i.e.,

i.e.,

i.e.,

i.e.,

i.e., , This is the required relqtion.

10.

Hence, (corresponding angles) ..... (i)

Material downloaded from myCBSEguide.com. 7 / 21


............ (ii)

Therefore, PQ = PR [sides opposite to equal angles ]

Hence, is isosceles.

11. Let C is the centre of two concentric circles of radii 5cm and 3 cm.

Let AB, a chord of the bigger circle touches the smaller circle at M.

CM = 3 cm, CA = 5 cm

[radius through point of contact is to tangent ]

[by Pythagoras Theorem ]

i.e.,

Therefore, AB=2AM = 2*4 = 8 cm.

[Line segment, drawn from centre of a circle perpendicular to any chord, bisects the chord ]

Hence, the length of the chord is 8 cm.

12. Let V, h and r denote the volume, height and base radius of the cylinder respectively.

Here, h=210 m,

Then,

Hence the volume of the cylinder is

Material downloaded from myCBSEguide.com. 8 / 21


SECTION - C

13. Let the fixed charge = ₹ x and the charge per km = ₹ y

According to the first condition, x + 10y = 200 …(i)

According to the second condition, x + 15y = 275 …(ii)

Now subtracting (i) from (ii), we get,

5y = 75

i.e., y = 15

From (i), x+ 10*15 = 200 (putting value of y ]

i.e., x = 200 - 150

i.e., x = 50

Hence, the fixed charge = 50 and the charge per km = ₹ 15

Now for travelling a distance of 25 km,

the person has to pay = ₹ (50 + 25*15) = ₹ 425.

14. let us assume that, is rational.

i.e., let for two coprime numbers a and b ( a, b are integers, b 0),

i.e.,

Since, is rational so, is rational.

This is a contradiction, because is irrational.

This contradiction is only for our incorrect assumption,

that, is rational.

Material downloaded from myCBSEguide.com. 9 / 21


Hence, the conclusion is that, is irrational.

15. According to Euclid’s division lemma,

Given positive integers a and b,

there exist unique integers q and r satisfying a = bq + r, 0 ≤ r < b.

Let us start with taking a, where a is a positive odd integer.

We apply the division algorithm with a and b = 4.

Since, 0 ≤ r < 4, the possible remainders are 0, 1, 2 and 3.

i.e., a can be 4q or 4q + 1 or 4q + 2, or 4q + 3, where q is the quotient.

However, since a is odd, a cannot be 4q or 4q + 2 (since they are both divisible by 2)

Therefore, any odd integer is of the form 4q + 1 or 4q + 3.

16. Let, a, d and be the first term, common difference and the term of the AP.

As,

According to the first condition,

i.e., a + (4 - 1)d = 3a

i.e., 3d = 2a …. (i)

Also, according to the 2nd condition,

i.e., a + (7 - 1)d - [a + (3 - 1)d ] = 1

i.e., a + 6d - a - 2d = 1

i.e., 4d = 1

i.e.,
Now from (i), i.e.,

Hence the first term = and common difference =

Material downloaded from myCBSEguide.com. 10 / 21


17. By joining A to C, we get and

As, area of a triangle with vertices at points

is

… (i)

So, using formula (i)

So the area of is .

Again, using formula (i),

So, the area of quadrilateral ABCD =

18. Let ABCD is rhombus.

Its diagonals AC and BD intersect at point O.

To prove that,

Material downloaded from myCBSEguide.com. 11 / 21


Since diagonals of a rhombus bisects each other perpendicularly,

so, AO = OC = ; BO = OD=

and AC is perpendicular to BD at point O.

and are all right angled triangle.

Using Pythagoras Theorem,

From ,

From ,

From ,

From ,

Adding (i), (ii), (iii), (iv), we get,

[ since AO = CO and BO = DO ]

i.e.,

Hence proved

19. Let from an external point P two tangents PQ and PR

are drawn to the circle with centre at O.

The tangents touch the circle at points Q and R.

We have to prove that, PQ = PR.

Let us draw the line segments OQ, OR and OP.

Now in DPOQ and DPOR,

Material downloaded from myCBSEguide.com. 12 / 21


[each = 90o as OQ and OR are radii

through points of contacts]

OQ = OR [radii of same circle]

OP = OP [ Common side]

Therefore, [RHS]

Hence, PQ = PR [corresponding part of congruent triangles]

20. We know,

21. One disc is drawn at random from the box means that all the discs are equally likely to be
drawn.

Let the event of drawing one disc bearing a perfect square number be E.

Material downloaded from myCBSEguide.com. 13 / 21


Given that total number of discs in the box = 90

Therefore, total number of all possible outcomes = 90

1, 4, 9, 16, 25, 36, 49, 64, 81 are perfect square numbers between 1 to 90. 1 Therefore,
outcomes favorable to the event E = 9

So,

22. One marble is taken out of the box at random means,

all the marbles are equally likely to be taken out. Therefore, the total number of possible
outcomes = 5 + 8 + 4 = 17

Let the event of taking out of one white marble be W

and also let the event of taking out of one green marble be G

Then number of outcomes favorable to the event W = 8

Therefore, [answer of (i) ]

Again the number of outcomes favourable to the event G = 4

So,

Therefore, P( not green) = 1 - P(G) [answer of (ii) ]

SECTION - D

23. Let the tap of smaller diameter can fill the tank separately in x hrs

Then the tap of bigger diameter can fill the tank separately in (x - 10) hrs.

According to the question,

two water taps together can fill a tank in hours i.e., in hrs.

Then, in 1 hr smaller diameter tap can fill part of the tank,

Material downloaded from myCBSEguide.com. 14 / 21


in 1 hr bigger diameter tap can fill part of the tank,

and 1 hr two taps together can fill part of the tank,

So,

i.e.,

i.e.,

i.e.,

i.e.,

i.e.,

Now, using the quadratic formula, we get,

Since , so, x = 25

Therefore, tap of smaller diameter can fill the tank separately in 25 hrs, and the tap of bigger
diameter can fill the tank separately in 15 hrs.

24. The first three-digit number which leaves remainder 3 when divided by 5 is 103.[as 103 =
5*20 + 3 ]

Last three-digit number which leaves remainder 3 when divided by 5 is 998. [ as 998 = 5*199
+ 3 ]

Now the three-digit numbers which leaves remainder 3 when divided by 5 are 103, 108, 111,
…, 998, which form an AP.

First term, a = 103, common difference, d = 108 - 103 = 5

Material downloaded from myCBSEguide.com. 15 / 21


Let, 998 be the term ( ) of the series.

As,

So,

i.e.,

i.e.,

Therefore, 998 is the 180th term of the series.

Now sum of the terms of the series

=99090

25. Steps of construction:

Draw any ray BX making an acute angle with BC on the side opposite to the vertex A.
Locate 4 (the greater of 3 and 4 in ) points on BX so that

Material downloaded from myCBSEguide.com. 16 / 21


Join B4C and draw a line through B3 (the 3rd point, 3 being smaller of 3 and 4 in )

parallel to B4C to intersect BC at C′.

Draw a line through C′ parallel to the line CA to intersect BA at A′.


Thus, A′BC′ is the required triangle

26.

Hence proved.

27. In ABC, , AB=3 units, AC = 4 units

So, by Pythagoras Theorem,

Material downloaded from myCBSEguide.com. 17 / 21


Area of the semi-circle on BC (as diameter)

i.e., area of BECB =

Area of the semi-circle on AC (as diameter)

i.e., area of ADCA =

Area of the semi-circle on AB (as diameter)

i.e., area of AFBA =

Also from the fig. (area of AHBA + area of AGCA )

= area of semi-circle BHGCB - area of ABC

= -

= -

Area of AFBHA + area of AGCDA

= area of AFBA + area of ADCA - (area of AHBA + area of AGCA )

= + - sq units.

Material downloaded from myCBSEguide.com. 18 / 21


Hence, area of the shaded part

i.e., area of area of BECB +area of AGCDA + area of AFBHA

= + + - sq. units

=+ + - sq. units

28. Let h meters be the height of the light-house AB.

Also let two ships be at C and D (in fig)

By question, CD = meters.

Angle of depression of two ships as observed

from the top (A) of the light-house are 60o and 45o respectively.

In the fig. ,

Therefore,

(alternate angle)

Now in ,

Again, in ,

Material downloaded from myCBSEguide.com. 19 / 21


[ by (i) ]

Hence, the height of the light-house is meters

29. Radius of the cylindrical tank = , its depth is 2 m.

Internal radius of the pipe =

So volume of the tank =

Flow of water through the pipe = 3 km/hr

Volume of water will flow through the pipe per minute

Therefore, time taken to fill the tank

So, flow of water should be stopped after 1 hr 40 min to avoid over flow from the tank.

Value: Water is most important substance in the earth for lives ( human, animals, plants) so

Material downloaded from myCBSEguide.com. 20 / 21


we should use our water wisely and not waste it.

30.

Class Interval Frequency Cumulative Frequency

0-10 2 2

10-20 3 5

20-30 5 10

30-40 6 16

40-50 x 16+x

50-60 3 19+x

60-70 2 21+x

Given median = 35. This lies in the class 30-40

So, l = 30, f = 6, cf = the cumulative frequency of the class preceding 30-40 = 10

We know,

Here,

Hence value of x is 5.

Material downloaded from myCBSEguide.com. 21 / 21


SAMPLE QUESTION PAPER 05
Class-X (2017–18)
Mathematics

Time allowed: 3 Hours Max. Marks: 80


General Instructions:
(i) All questions are compulsory.
(ii) The question paper consists of 30 questions divided into four sections A, B, C and D.
(iii) Section A contains 6 questions of 1 mark each. Section B contains 6 questions of 2 marks
each. Section C contains 10 questions of 3 marks each. Section D contains 8 questions of 4
marks each.
(iv) There is no overall choice. However, an internal choice has been provided in four
questions of 3 marks each and three questions of 4 marks each. You have to attempt only one
of the alternatives in all such questions.
(v) Use of calculators is not permitted.

SECTION – A

1. Which term of the progression 4, 9, 14, 19, … is 109 ?

2. Find the sum of the zeroes of the quadratic polynomial .

3. If and , then what is the value of cot a ?

4. A die is thrown once. What is the probability of getting a composite number?

5. Find the coordinates of a point A, where AB is the diameter of a circle whose centre is (2, –
3) and B is (1, 4).

6. In fig-1, AQ, AR and BC are the tangents. If AQ = 10 cm, find the perimeter of DABC.

Material downloaded from myCBSEguide.com. 1 / 20


SECTION – B

7. Determine an A.P whose 3rd term is 16 and difference of 7th term and 5th term is 12.

8. Find the coordinates of the points of trisection of the line segment joining points (4, –1) and
B(–2, –3).

9. Find two consecutive positive integers, sum of whose squares is 365.

10. In Fig–2, and AB = 6 cm. Find the length of DC.

11. In fig-3, from an outside point P, PA is a tangent to a circle whose centre is at C. A is the
point of contact. If PC = 10 cm and PA= 8 cm. Find the diameter of the circle.

12. 2 cubes each of volume are joined end to end. Find the surface area of the
resulting cuboid.

SECTION – C

Material downloaded from myCBSEguide.com. 2 / 20


13. Find two consecutive odd positive integers, sum of whose squares is 970.

14. How many terms of the sequence 18, 16, 14, … should be taken so that their sum is zero?

15. In fig-4, DE || BH and EF || HC. Show that, DF || BC.

16. Show that the points (1, 7), (4, 2), (–1, –1) and (– 4, 4) are the vertices of a square.

Or

Find the area of the rhombus, if its vertices are (3,0), (4,5), (-1,4) and (-2,-1) taken in order.

17. The ages of workers in a factory are given below :

Age (in years) 20-30 30-40 40-50 50-60 60-70

Number of workers 5 26 78 104 98

Find the modal age of workers.

Or

Compute the Median for the given data.

Class –interval 100-110 110-120 120-130 130-140 140-150 150-160

Frequency 6 35 48 72 100 4

18. Two dice one blue and one grey, are thrown at the same time. What is the probability
that the sum of the two numbers appearing on the top of the dice is 6?

19. Show that is irrational.

Material downloaded from myCBSEguide.com. 3 / 20


Or

Use Euclid’s division algorithm to find the HCF of 4052 and 12576.

20. Prove that,

21. In fig–5 ABCD is a quadrant of a circle with radius 28 cm and a semicircle BEDB is drawn
with BD as diameter. Find the area of the shaded region.

Or

The radius of a radius of a circle is 20cm Three more concentric circles are drawn inside it in
such a manner that it is divided into four parts of equal area. Find the radius of the largest of
the three concentric circle

22. Find the HCF of 96 and 404 by the prime factorization method. Hence, find their LCM.

SECTION – D

23. Construct a tangent to a circle of radius 4 cm from a point on the concentric circle of
radius 6 cm.

24. Five years hence, the age of Anubhab will be three times that of his son. Five years ago,
Anubhab’s age was seven times that of his son. What are their present ages?

25. Prove that: .

Or

If a cosθ – b sin θ = c prove that

26. The king, queen and jack of clubs are removed from a deck of 52 playing cards and the
remaining cards are shuffled. A card is drawn from the remaining cards. Find the

Material downloaded from myCBSEguide.com. 4 / 20


probability of getting a card of (i) heart (ii) king (iii) club.

27. In fig-6, AB and CD are two diameters of a circle with center at O. . OD is the
diameter of the smaller circle.Given that, OA = 7 cm, find the area of the shaded region.

28. From the top of a building 100 m. high, the angles of depression of the top and bottom of

a tower are observed to be 45o and 60o respectively. Find the height of the tower. Also find
the distance between the foot of the building and the bottom of the tower.

29. A wooden article was made by scooping out a hemisphere from each end of a solid

cylinder, as shown in fig-7. If the height of the cylinder is 10 cm and its base is of radius 3.5
cm, find the total surface area of the article.

30. The sum of first six terms of an A.P is 42. The ratio of its 10th term to its 30th term is 1 : 3.

Find the 1st term and the 13th term of the A.P.

Or

For what values of n nth term of the series 3, 10, 17...And 63, 65, 67..Are equal.

Material downloaded from myCBSEguide.com. 5 / 20


CBSE SAMPLE PAPER 05
CLASS X - Mathematics
Marking Scheme

SECTION – A

1. 4+(n – 1).5 = 109

(n – 1).5 = 105

n = 21 + 1=22

2.

are the two zeroes.

So, sum the zeroes =

3. Given =

4. From 1 to 6 there are two composite numbers 4 and 6.

P(getting a composite number)

5. Let (x, y) be the coordinates of point A.

Material downloaded from myCBSEguide.com. 6 / 20


Coordinates of A are (3, –10).

6. Perimeter of ABC =AB+BC+AC

= (AQ – QB)+(BS + SC)+(AR – RC)

=AQ – BS + BS + RC +AQ – RC = 2AQ = 20 cm.

[Since The lengths of tangents drawn from an external point to a circle are equal.

SECTION – B

7. n-th term, , a = 1st term and d = common difference of the AP

Now d = 6

Also, a+ (3 – 1)d = 16 or, a + 12 = 16 or, a = 4.

Therefore the terms of the AP are 4, 10, 16, 22, 28, …

8. Let point P(x, y) trisects the line segment joining points (4, –1) and B(–2, –3) [P is nearer to
the first point (4, –1) ]

Then, coordinates of P are

Again let point Q(x, y) trisects the line segment joining points (4, –1) and B(–2, –3)

Material downloaded from myCBSEguide.com. 7 / 20


[Q is nearer to the second point (–2, –3)]

Then, coordinates of Q are

9. Let the consecutive positive numbers be x and (x+1)

According to the question,

x = 13, since x is positive,


Hence the required numbers are 13 and 14

10. We know that, if one angle of a triangle is equal to one angle of the other triangle and the
sides including these angles are proportional, then the two triangles are similar.

Here, [vertically opposite]


Given,

Material downloaded from myCBSEguide.com. 8 / 20


Therefore, ~ .
i.e.,

Hence, i.e., DC = 12 cm.

11. We know that, The tangent at any point of a circle is perpendicular to the radius through
the point of contact.

So, .

By Pythagoras theorem,

Hence the diameter of the circle = 2*6 cm = 12 cm.

12. Length of each side of the cube of volume is 4 cm. If 2 cubes each of volume
are joined end to end, then a cuboid will be formed. The dimensions of the cuboid
formed are (4 + 4) cm, 4 cm , 4 cm i.e., 8 cm, 4 cm and 4 cm. The surface area of the resulting
cuboid

SECTION – C

13. Let the consecutive odd positive integers be (2x – 1) and (2x + 1). (x is positive integer
1]

According to question,

i.e.,

Material downloaded from myCBSEguide.com. 9 / 20


Hence the required numbers are (2 11 – 1) and (2 11 + 1) i.e., 21 and 23.

ALTERNATE PROCESS:

Let the consecutive odd positive integers be x and (x + 2) [ x 1 ]

According to question,

[since, x is odd positive integer]

x = 21.

Hence the required numbers are 21 and 23.

14. In the given sequence, first term, a = 18,common difference, d = 16 – 18 =14 – 16 = – 2 =


constant Therefore, the sequence is an AP.

Let the sum of the first n terms is zero. We know, .

i.e.,

Hence, the sum of 19 terms of the sequence is zero.

Material downloaded from myCBSEguide.com. 10 / 20


15. By question DE || BH.

Therefore in ABH,

[Since, any line, drawn parallel to one side of a triangle, divides the other two sides in the
same ratio.]

Also by question, EF || HC.

So, [by same reason ]

Therefore by (i) and (ii),

We know that, if a line divides any two sides of a triangle in the same ratio, then the line is
parallel to the third side.

DF || BC [ by (iii)]
Hence proved.

16. Let (1, 7), (4, 2), (–1, –1) and (– 4, 4) are the coordinates of the points A, B, C and D
respectively.

Length of ,

Length of

Length of

Length of

Material downloaded from myCBSEguide.com. 11 / 20


Now,

Since, the lengths of the sides are equal, so ABCD is either a square or a rhombus.

Length of

Length of

Again,

i.e., lengths of the diagonals are equal. So ABCD is a square.

Hence, the given points are the vertices of a square.

Or

Let (3, 0), (4, 5), (−1, 4) and (−2, −1) are the vertices A, B, C, D of a rhombus ABCD

17. From the given data, we se that the maximum frequency is 104 and the corresponding
class is 50-60.

Material downloaded from myCBSEguide.com. 12 / 20


Therefore, the modal class is 50-60.

Here, l = 50, h = 10, f = 104, and

Therefore,

i.e.,

Hence the modal age of workers is 58.125 years.

18. When the blue die shows 1, the grey die can show any one of the numbers 1, 2, 3, 4, 5, 6.
The same is true when the blue die shows 2, 3, 4, 5, 6. So, the number of possible outcomes =
6 × 6 = 36.
Let the event that, ‘the sum of the two numbers appearing on the top of the dice is 6’ be
denoted by E.

Thereforem the outcomes favourable to the event E are:

(1, 5), (2, 4), (3, 3), (4, 2), (5, 1)

So number of outcomes favourable to the event E = 5

Therefore,

19. If possible let us assume, that, is a rational number.

i.e., , where p and q are coprime numbers and

i.e.,

As, 5, p, q are all integers so is rational.

Now from (i) we get that is also a rational.

Material downloaded from myCBSEguide.com. 13 / 20


But this contradicts the fact that is always an irrational number.

Therefore our assumption is wrong.

Hence we conclude that, is irrational.

20.

.
Hence proved.

21. Since ABCD is a quadrant of a circle with radius 28 cm,

so, AB = AD = 28 cm.

Area of the quadrant ABCD = sq. cm

area(BDCB) = area(quadrant ABCD) – area ( ABD)

= sq. cm

By Pythagoras Theorem,

Now, area of the shaded region = area (semi-circle BEDB) – area(BDCB)

Material downloaded from myCBSEguide.com. 14 / 20


=

= sq. cm

22. By prime factorization, of 96 and 404 can be written as

and

Therefore, the HCF of 96 and 404 is

Since Product of two numbers = (their HCF) (their LCM)

Hence LCM of 96 and 404 =

SECTION – D

23.

Steps of construction :

Material downloaded from myCBSEguide.com. 15 / 20


Draw a circle of 4 cm radius with centre as O.
Draw a circle of 6 cm radius taking O as its centre.
Locate a point P on this circle and join OP.
Bisect OP. Let M be the mid-point of OP.
Take M as its centre and MO as its radius, draw a circle. Let it intersect the given circle
at the points Q and R.
Join PQ and PR.
PQ and PR are the required tangents.

24. Let the present age of Anubhab and his son be x years and y years respectively. Five
years hence, age of Anubhab will be (x+5) years and five years hence his son’s age will be
(y+5) years. By question, (x+5) = 3(y+5)
i.e., x – 3y = 15– 5
i.e., x – 3y = 10 … (i)
Five years ago, Anubhab’s age was (x–5) years and that of his son was (y – 5) years.

By question, (x–5)=7(y – 5)
i.e., x – 7y = 5–35
i.e., x – 7y = –30 … (ii)

Now subtracting (ii) from (i),

From (i), x= 10 + 3 10 = 40

Hence, the present ages of Anubhab and his son are 40 years and 10 years.

25.

[ Multiplying numerator and denominator by (cosecA – cotA) ]

Material downloaded from myCBSEguide.com. 16 / 20


[multiplying numerator and denominator by (1-cosA)]

[ Dividing numerator and denominator by sinA ]

Hence proved.

26. After removing king, queen and jack of clubs from a deck of 52 playing cards there are 49
cards left in the deck. Out of these well shuffled 49 cards one card can be chosen in 49
different ways. Therefore number of total outcomes = 49

i) There are 13 heart cards in the deck of well shuffled 49 cards out of which one heart card
can be chosen in 13 different ways. Therefore, number of outcomes favourable to the event
of getting heart = 13

P(getting one heart card ) =

ii) There are 3 king cards in the deck of well shuffled 49 cards out of which one king can be
chosen in 3 different ways.

P(getting one king ) =

iii) There are (13 – 3) i.e., 10 club cards in the deck of well shuffled 49 cards out of which one
club can be chosen in 10 different ways.

P(getting one club card ) =

27. According to question, OA= 7 cm.

Therefore, OA = OB = OD = 7 cm.

Material downloaded from myCBSEguide.com. 17 / 20


Area of the circle with OD as diameter

Area of the semi-circle with OA as radius

Area of ABC =

Hence the area of the shaded area

= Area (circle with OD as diameter) + area(semi-circle with OA as radius) – ar( ABC)

28. Let AB and CD are the building and the tower respectively.

By question, AB = 100 m

Also by question, angles of depression of the top and bottom of a tower are observed from

the top of the building to be 45o and 60o respectively.

In the figure, .

Let, the height of the tower CD be h meter.

is drawn. EB = CD = h meter and AE = (100 – h) m

Material downloaded from myCBSEguide.com. 18 / 20


In DAEC, (alternate angle)

Then, in DAEC, i.e., EC = AE = (100 – h) m

BD= EC = (100 – h) m

Now in DABD, (al)

then,
i.e.,

i.e.,

i.e.,

Hence the height of the tower = 42.27 m (approx.)

The distance between the foot of the building and the bottom of the tower = 100 – 42.27 =
57.73 m

29. Let r and h are the base radius and height of the cylinder respectively.

Given, h = 10 cm and r = 3.5 cm.

Then radius of each of the hemisphere = 3.5 cm.

We know that, curved surface are of a cylinder = sq. units

and surface area of the hemisphere = sq. units

Material downloaded from myCBSEguide.com. 19 / 20


Total surface area of the article = Curved surface area of the cylinder + 2 (surface area of
one hemisphere)

= sq. units

sq. units

30. We know that, if a and d are the first term and the common difference of an AP, then,
term, and sum of first n terms,

According to the question,

i.e.,

i.e.,

Also by question,

i.e.,

i.e.,

i.e.,

i.e.,

i.e.,

Now from (i), 7d = 14, i.e., d = 2

a = d = 2

Now 13th term of the AP = 2+ (13 – 1)2 = 26

Hence the first term = 2 and the 13th term = 26.

Material downloaded from myCBSEguide.com. 20 / 20


SAMPLE QUESTION PAPER 06
Class-X (2017–18)
Mathematics

Time allowed: 3 Hours Max. Marks: 80


General Instructions:
(i) All questions are compulsory.
(ii) The question paper consists of 30 questions divided into four sections A, B, C and D.
(iii) Section A contains 6 questions of 1 mark each. Section B contains 6 questions of 2 marks
each. Section C contains 10 questions of 3 marks each. Section D contains 8 questions of 4
marks each.
(iv) There is no overall choice. However, an internal choice has been provided in four
questions of 3 marks each and three questions of 4 marks each. You have to attempt only one
of the alternatives in all such questions.
(v) Use of calculators is not permitted.

SECTION - A

1. Find the coefficient of x0 in x2+ 3x + 2 = 0

2. What is the common difference of the A.P. in which a18-a14=32?

3. If the points A(x, 2), B(-3,-4) and C(7, -5) are colinear, then find the value of x.

4. If PA and PB are tangents from an outside point P such that PA = 15 cm and APB = 60°.
Find the length of chord AB.

5. Find the length of the shadow of a 20m tall pole, on the ground when the sun’s elevation is
45° .

6. If the probability of winning a game is 0.995, then find the probability of losing a game.

SECTION - B

7. Solve: .

Material downloaded from myCBSEguide.com. 1 / 19


8. Find the number of terms in the following series:

-5 + (-8) + (-11) + ……… + (-230)

9. Find the area (in sq.units) of the triangle formed by the points A(a, 0), O(0, 0) and B(0, b).

10. In two similar triangles ABC and PQR, if their corresponding altitudes AD and PS are in
the ratio 4 : 9, find the ratio of the areas of triangles ABC and PQR.

11. In the adjoining figure, O is the centre of the circle. AP and AQ two tangents drawn to the
circle. B is a point on the tangent QA and PAB = 125°, Find POQ.

12. The dimensions of a metallic cuboid are: 100 cm, 80 cm, 64 cm. It is melted and recast into
a cube. Find the surface area of the cube.

SECTION - C

13. The two forces are acting on a body such that their maximum and minimum value of
resultant is 27 N and 13 N respectively. Find the values of forces.

14. Prove that is irrational.

15. A sweet seller has 420 Kaju burfis and 130 Badam barfis. She wants to stack them in such
a way that each stack has the same number, and they take up the least area of the tray. What
is the maximum number of burfis that can be placed in each stack for this purpose? (Use
Euclid division algorithm)

16. Find the sum of all multiples of 9 lying between 100 and 500.

Or

Material downloaded from myCBSEguide.com. 2 / 19


Check whether 301 is a term of the list of numbers 5, 11, 17, 23, . . .

17. Find a point on y axis which is equidistant from (2, 2) and (9, 9).

Or

Find a relation between x and y if the points (2, 1) , (x, y) and (7, 5) are collinear

18. The hypotenuse of a right triangle is 6 m more than the twice of the shortest side. If the
third side is 2 m less than the hypotenuse, then find the sides of the triangle.

19. Prove that the parallelogram circumscribing a circle is a rhombus.

Or

An umbrella has 8 ribs, which are equally spaced, a ssuming umbrella to be a flat circle of
radius 45 cm. Find the area between two c onsecutive ribs of the umbrella.

20. If 7 cosec A - 3cot A = 7, prove that 7cot A - 3cosec A = 3.

21. A card is drawn from a well- shuffled pack of 52 playing cards. What is the probability
that the card drawn is
(a) either a red or a king
(b) a black face card
(c) a red queen card.

Or

At a car park there are 100 vehicles, 60 of which are cars, 30 are vans and the remainder are
lorries. If every vehicle is equally likely to leave, find the probability of:
(a) van leaving first.
(b) lorry leaving first.
(c) car leaving second if either a lorry or van had left first

22. From a bag containing 5 red, 6 black and 7 yellow balls, a ball is drawn at random. Find
the probability that it is:
(a) not a yellow ball
(b) neither a black nor a red ball

Material downloaded from myCBSEguide.com. 3 / 19


(c) either a black or a yellow ball

SECTION - D

23. The sum of the areas of two squares is 640 m2. If the difference in their perimeters be
64m find the sides of the two squares.

Or

In a class test, the sum of marks obtained by Puneet in Mathematics and Science is 28. Had
he got 3 more marks in Mathematics and 4 marks less in Science, the product of marks
obtained in the two subjects would have been 180? Find the marks obtained in two subjects
separately.

24. A girl walking uphill covers a distance of 20 m during the first minute, 18m during the
second minute and 16m during the third minute and so on. How much distance will she

cover in (i) 10th minute (ii) 10 minutes?

25. Draw a circle of radius 8 cm. From a point 12 cm away from its centre, construct the pair
of tangents to the circle and measure their lengths.

Or

Draw a line segment AB of length 8 cm. Taking A as centre, draw a circle of radius 4 cm. and
constant the pair of tangents of the circle from point B and measure their lengths.

26. If sec A+ tan A= p, then prove that sin A = (p2+1)/(p2-1)

27. In the adjoining figure, PQ=24 cm, PR=7 cm and O is the centre of the circle. Find the area
of the shaded region.

Material downloaded from myCBSEguide.com. 4 / 19


28. From the top of a hill, the angles of depression of two consecutive kilometre stones due
east are found to be 30° and 45°. Find the height of the hill.

29. 21 glass spheres each of radius 2 cm are packed in a cuboidal box of internal dimensions
16 cm x 8 cm x 8 cm and then the box is filled with water. Find the volume of water filled in
the box.

30. The following frequency distribution gives the monthly consumption of electricity of 68
consumers of a locality. Find the mean, median and mode of the data.

Monthly Consumption 65-85 85-105 105-125 125-145 145-165 165-185 185-205

Number of Consumers 4 5 13 20 14 8 4

Or

The median of the following dats is 525. Find the value of x and y if the total frequencry is
100.

0- 100- 200- 300- 400- 500- 600- 700- 800- 900-


Classes
100 200 300 400 500 600 700 800 900 1000

Number of
2 5 X 12 17 20 Y 9 7 4
Consumers

Material downloaded from myCBSEguide.com. 5 / 19


CBSE SAMPLE PAPER 06
CLASS X - Mathematics
Solutions

Section A

1. 2

2. d=8

3. -63

4. AB=15cm

5. 20cm

6. 0.005

Section B

7.

8. a = -5 ; d = -3

an = -230

-230= -5 + (n – 1) (-3)

=> -225 = (n-1)(-3)

=> 75 = n-1
=> n = 76

Material downloaded from myCBSEguide.com. 6 / 19


9. Area of ABC =

= - sq.units

= sq.units ( as area cannot be negative)

10. Since the areas of two similar triangles are in the ratio of the squares of corresponding
altitudes,

Thus, the ratio of areas of triangles ABC and PQR = 16 : 81

11. PAB+ PAQ =180° [Linear Pair of angles]

PAQ + 125o = 180o

PAQ = 180o - 125o

PAQ = 55o

In Quadrilateral APOQ,

PAQ+ APO+ AQO+ POQ = 360°

55°+90°+90°+ POQ=360° [Radius is perpendicular with tangent at their point of contact]

POQ=360°- 235°

POQ =125°.

12. Volume of cube = Volume of cuboid

(side)3 = Length × Breadth × Height

(side)3 = 100×80×64

Material downloaded from myCBSEguide.com. 7 / 19


Side =

= 80 cm.

Surface Area of the Cube = 6a2 = 6 × 80 × 80 = 38400 cm2

Section C

13. Let the two forces in Newton(N) are x and y respectively.

Then,

x + y = 27 ------(1) [ For maximum force]

x - y = 13 -------(2) [ For minimum force]

Solving (1) and (2)

x = 20 ; y = 7

Required forces are 20N & 7N

14. Suppose is rational.

p and 1 are coprime

But is irrational while is rational and an irrational can never be equal to a rational
number.

Thus our assumption is wrong and hence is irrational.

15. The number of stacks will be least if the number of burfis in each stack is equal to the
HCF of 420 and130.
Now, let us use Euclid’s algorithm to find their HCF.

420 = 130 × 3 + 30

130 = 30 × 4 + 10

Material downloaded from myCBSEguide.com. 8 / 19


30 = 10 × 3 + 0

So, the HCF of 420 and 130 is 10.

Hence, the sweet seller can make stacks of 10m burfis of each kind to cover the least area of
the tray.

16. a = 108 , an = l = 495 ; d = 9

an = a + (n – 1) d

495 = 108 + (n -1) 9

n = 44

S44 = (a + l)

= (108 + 495)

= 13,266

Or

We have : a2 – a1 = 11 – 5 = 6,

a3 – a2 = 17 – 11 = 6,

a4 – a3 = 23 – 17 = 6

As ak + 1 – ak is the same for k = 1, 2, 3, etc.,

the given list of numbers is an AP.


Now, a = 5 and d = 6.

Let 301 be a term, say, the nth term of the this AP. We know that
an = a + (n – 1) d

So, 301 = 5 + (n – 1) × 6
i.e., 301 = 6n – 1 So,
n =
But n should be a positive integer.
So, 301 is not a term of the given list of numbers.

Material downloaded from myCBSEguide.com. 9 / 19


17. PA = PB

Squaring on both the sides,

Point is (0,11)

Or

If the given points A(2,1), B(x,y) and C(7,5) are collinear, then the area formed by these
points will be 0.

This is the requried relation between x and y.

18. Let the shortest side be x meters in length.

Then, hypotenuse = (2x+16)m

And the third side = (2x+4)m

Material downloaded from myCBSEguide.com. 10 / 19


By using Pythagoras theorem, we have

(2x + 6)2 = x2 + (2x + 4)2

(x – 10)(x + 2) = 0

x = 10 or x = -2

19. Let ABCD be parallelogram circumscribing circle with centre 0.

AS = AP (Tangents from A)

BP = BQ (Tangents from B)

CQ = CR (Tangents from C)

DS = DR (Tangents from D)

Now, AP + BP + CR + DR = AS + BQ + CQ + DS

( AP + BP) + ( CR + DR) = (AS + DS ) + ( BQ + CQ )

AB + CD = AD + BC

2 AB = 2 BC

AB = BC

AB = BC = CD = DA

ABCD is a rhombus

20. 7cosecA – 3 cotA = 7

Material downloaded from myCBSEguide.com. 11 / 19


7cosecA – 7 = 3 cotA

7(CosecA – 1) = 3 cotA

Multiplying by (cosecA + 1) both sides, we get

7(cosecA – 1) (cosecA + 1) = 3 cotA (cosecA + 1)

7(cosec2A-1) = 3 cotA (cosecA + 1)

7cot2A=3 cotA (cosecA + 1)

7cotA = 3(cosecA + 1)

7cotA – cosecA=3

21. n(S)= 52

(a). Let A be the event of drawing either a red or a king card

n(A) = 28

P(A) =

(b) Let B be the event of drawing a black face card.

n(B) = 6

P(B) =

(c) Let C be the event of drawing a red queen card.

n(C) = 2

P(C) =

22. (a) let A be the event of drawing a ball other than yellow in colour.

n(A) = 11

P(A) =

Material downloaded from myCBSEguide.com. 12 / 19


(b). Let B be the event of drawing a ball which is yellow in colour.

n(B) = 7

P(B) =

(c). Let C be the event of drawing a ball which is either black or yellow .

n(C) = 13

P(C) =

Section D

23. Let the areas of two squares be a12 , a22

a12+ a22 = 640 --------------------(1)

4 a1-4a2 =64

a1 – a2 = 16

a1 = 16 +a2

Substitute the value of a1 in (i) we get

(16 +a2)2 + a22 = 640

a22 + 16a2 -192 = 0

Material downloaded from myCBSEguide.com. 13 / 19


= 8, -24

But length cannot be negative.

a2 = 8

a1 = 24

Therefore the sides are 24 & 8.

24. The required sequence is 20,18,16,14, ..........

a = 20, d = -2, n = 10

(i). 10th minute

an =a + (n -1) d

a10 =20 + 9(-2)

= 2 m

She covers a distance of 2 m during 10th minute.

(ii) 10 minutes

= 5(22)

= 110

After 10 minutes she will cover 110 m.

Material downloaded from myCBSEguide.com. 14 / 19


Steps of Construction:

Step 1: Draw a circle with centre O and radius 8 cm.

Step 2: Take a point P outside the circle such that OP = 12 cm. Join OP.

Step 3: Draw a perpendicular bisector of line segment OP. It intersects OP at O’.

Step 4: Take O’ as centre and O’O as radius and draw a circle which intersects the previous
circle at points A and B.

Step 5: Join P to A and P to B.

PA & PB are the required tangents from point P to circle with centre O.

Length of PA = PB = 14.4 cm

26. Let p (sec A + tan A)

By componendo and dividend method.

Material downloaded from myCBSEguide.com. 15 / 19


sinA = LHS

27. PQ = 24 cm, PR = 7 cm

Since P is a right angle, By Pythagoras theorem, we have

QR2 = PQ2 + PR2

= 242 + 72

= 576 + 49

= 625

QR = 25

Since QR is a diameter of a circle, radius of the circle r = 25/2 cm.

Area of the shaded region = Area of semi circle – Area of right triangle PQR

28. Let h = height of the hill

C and D are position of two stones.

Material downloaded from myCBSEguide.com. 16 / 19


In CAB, we have

In DAB, we have

29. Given, Radius of 1 glass sphere. R = 2cm

Therefore, Volume of such 21 glass spheres

Given, Dimensions of the cuboidal box = 16 cm × 8 cm × 8 cm

Therefore, Volume of the cuboidal box = 16 × 8 × 8

Volume of the water added in cuboid box =

Material downloaded from myCBSEguide.com. 17 / 19


Volume of the cuboidal box – volume of the 21 glass sphere

= 320 cm3

30.

Monthly No. of
Cumulative Class
consumption (in consumers di=xi-135 ui fiui
frequency(cf) mark
units) (fi)

65-85 4 4 75 -60 -3 -12

85-105 5 9 95 -40 -2 -10

105-125 13(f0) 22 115 -20 -1 -13

125 - 145 20(f1) 42 135 0 0 0

145-165 14(f2) 56 155 20 1 14

165-185 8 64 175 40 2 16

185 - 205 4 68 195 60 3 12

TOTAL 68 7

From the table, n = 68, n/2 = 34, h = 20

Mean =

Material downloaded from myCBSEguide.com. 18 / 19


Thus, the mean monthly consumption of electricity is 137.05 units.

Now, 125 - 145 is the median class because its frequency 42 is just greater than 34.

Median =

= 125 + 12

= 137 units

Model class is 125 – 145 because it has highest frequency

Mode =

= 125 + 10.76

= 135.77.

Material downloaded from myCBSEguide.com. 19 / 19


SAMPLE QUESTION PAPER 07
Class-X (2017–18)
Mathematics

Time allowed: 3 Hours Max. Marks: 80


General Instructions:
(i) All questions are compulsory.
(ii) The question paper consists of 30 questions divided into four sections A, B, C and D.
(iii) Section A contains 6 questions of 1 mark each. Section B contains 6 questions of 2 marks
each. Section C contains 10 questions of 3 marks each. Section D contains 8 questions of 4
marks each.
(iv) There is no overall choice. However, an internal choice has been provided in four
questions of 3 marks each and three questions of 4 marks each. You have to attempt only one
of the alternatives in all such questions.
(v) Use of calculators is not permitted.

Section A

Q.1 Is x = 2. y = 3 a solution of linear equation ?

Q.2 The nth term of an AP is 7-4n. Find the common difference

Q.3 Find the value of .

Q.4 A ladder is 10 meter long reaches a window 8 meter above the ground. Find the distance
of the foot of the ladder from the base of the wall.

Q.5 What is the relation between the lengths of two tangents from an external point to a
circle?

Q.6 One card is drawn from well shuffled pack of 52 cards. Find the probability that the card
will be ace.

Section B

Material downloaded from myCBSEguide.com. 1 / 32


Q.7 Find the highest positive integer by which dividing the numbers 396, 436, 542 reminders
5, 11, and 15 respectively.

Q.8 For what value of p, the following system of equation have a unique solution :

Q.9 Find two numbers whose sum is and product is

Q.10 If then evaluate

Q.11 The shadow of tower standing on a plane ground is found to be 40 m longer when the
Sun’s altitude reduces to from . Find the height of the tower.

Q.12 A hemispherical tank full of water id emptied by a pipe at the rate of litre per
second, How much time will take to empty the full tank if it is 3m in diameter.

Section C

Q.13 A pole has to be erected at a point on this boundary of a circular part of diameter 13
meters in such a way that the difference of its distances from two diametrically opposite
fixed gates A and B on the boundary is 7 meters. Is it possible to do so? If yes, at what
distance from the two gates should the pole be erected.

Q.14 Find the sum of number between 1 to 100 divisible by 6.

Q.15 In the given figure, the line segment XY is parallel to side AC of and it divides
the triangle into two parts of equal areas. Find the ratio .

Material downloaded from myCBSEguide.com. 2 / 32


Q.16 Find the area of triangle formed by joining the mid points of the sides of the triangle
ABC whose vertices are A(0,-1), B(2,1) and C(0,3). Find the ratio of this area and the area of
ABC.

Or

Find the cirumcentre of the triangle whose vertices are (-2, -3), (-1, 0), (7, -6).

Q.17 From a point on a bridge across a river the angles of depression of the banks on
opposite sides of the river are and respectively. If the bridge is at a height of 4m
from the banks, find the width of the river.

Or

If the angle of elevation of a cloud from a point 'h' metres above a lake is α and the angle of
depression of its reflection in the lake is β, prove that the height of the cloud is

Q.18 Two concentric circles are if radii 25 cm. and 7 cm. respectively. Find the length of the
chord of the bigger circle which touches the smaller circle.

Q.19 Draw a line segment AB of length 8 cm. Taking A as centre, draw a circle of radius 4 cm.
and constant the pair of tangents of the circle from point B and measure their lengths.

Or

Draw a circle of radius 8 cm. From a point 12 cm away from its centre, construct the pair of
tangents to the circle and measure their lengths.

Q.20 Find the co- ordinate of the circumcentre of the triangle whose vertices are (8,6), (8,-2)
and (2, -2). Also its circum radius.

Q.21 The distribution below shows the pocket money during a days of 60 employees in an
office. Find the median pocket money of the employees.

Pocket Money (in Rupees) 5-10 10-15 15-20 20-25 25-30 30-35 35-40

Number of Employees 3 6 9 11 14 10 7

Material downloaded from myCBSEguide.com. 3 / 32


Or

A survey conducted on 20 households in a locality by a group of students resulted in the


following frequency table for the number of family members in a household. Find the mode.

Family size 1-3 3-5 5-7 7-9 9-11

No. of families 7 8 2 4 1

Q.22 A box contains 30 discs which are number 1 to 30. If one disc is drawn at random from
the box. Find the probability that it hears –
(i) A two digit number
(ii) A perfect square number.

Section D

Q.23 Prove that is an irrational number.

Or

Use Euclid’s division lemma to show that the cube of any positive integer is of the form 9m,
9m + 1 or 9m + 8.

Q.24 Divide by and verify the division algorithm.

Q.25 Coach of a cricket team buys one bat and 2 balls for Rs. 300.Later he buys another 2
bates and 3 balls of the same kind for Rs. 525. Represent this situation algebraically and solve
it by graphical method. Also find out that how much money coach will pay for the purchase
of one bat and one ball.

Or

The sum of two numbers is 16 and the sum of their reciprocals is . Find the numbers.

Q.26 BE and CF are medians of triangle ABC right angled at A. Prove that 4(BE2 + CF2) = 5BC2

Q.27 Prove that

Material downloaded from myCBSEguide.com. 4 / 32


Q.28 Two circular flower beds lie on two sides AB and CD of a square lawn ABCD of side 56
m. If the centre of each circular flower bed is point of intersection O of the diagonals of
square lawn. Find the sum of the areas of the lawn and flower beds.

Q.29 A tent is the form of a cylinder of diameter 4.2 m and height 4 m, surrounding by a cone
of equal base and height 2.8 m. find the capacity of the tent and the cost of canvas for making
the tent at 100 per sq. m.

Q.30 The median of the following dats is 525. Find the value of x and y if the total frequencry
is 100.

0- 100- 200- 300- 400- 500- 600- 700- 800- 900-


Classes
100 200 300 400 500 600 700 800 900 1000

Number of
2 5 X 12 17 20 Y 9 7 4
Consumers

Or

The distribution below gives the marks of 100 students of a class.

Marks 0-5 2-10 10-15 15-20 20-25 25-30 30-35 35-40

No. of Students 4 6 10 10 25 22 18 5

Draw a less than type and a more than type ogive from the given data. Hence, obtain the
median marks from the graph.

Material downloaded from myCBSEguide.com. 5 / 32


CBSE SAMPLE PAPER 07
CLASS X - Mathematics
Solutions

Sol.1 The given equation is


Putting in given equation, we have

=
=
=
Hence, is a solution of given equation.

Sol.2 Here, an=7-4n

If n=1 then a1=7-4x1=3

If n=2 then a2=7-4x2=-1

Common difference d= a2- a1=-1-3=-4

Sol.3 .(putting the value of trigonometric ratios)

Sol.4 According to the figure ABC is a right angled triangle in which


Now, By the Pythagoras Theorem

Material downloaded from myCBSEguide.com. 6 / 32


Sol.5 Both tangent lines are equal.
Sol.6 There are 4 aces in a pack

The number of Favorable outcomes=4

The total number of outcomes=52

P (card is ace)

Sol.7 It is given that on dividing 396 by the required number, there is remainder of 5; this
means that 396-5 = 391 is exactly divisible by the required number.

Similarly 436 -11 = 425 is also exactly divisible by the required number and

542 – 15 = 527 is also exactly divisible by required number.

Also the required number is the largest number satisfying the above property.

Therefore it is HCF of 391, 425 and 527.

Let us now find the HCF of 391, 491 and 527

391 = 17 23

425 = 5 5 17

527 = 17 31

HCF (391, 425, 527) = 17

Sol.8 For unique solution

Or

Sol.9 Let one number be x

Material downloaded from myCBSEguide.com. 7 / 32


Anotthe number will be (27-x ) [ Sum of two number = 27]

Their product

According to the question

So the required two numbers are 13 and 14 Ans.

Sol.10

Sol.11 Let AB be height of the tower and BC m

Material downloaded from myCBSEguide.com. 8 / 32


DB m

In ABC, we have

In ABD, we have

Substituting obtained from equation in equation

We get

Material downloaded from myCBSEguide.com. 9 / 32


Sol.12 radius of hemispherical tank m

Volume of the tank

So, the volume of water to be emptied litres

Litres

Since, litres of water is emptied in 1 second


litres of water will be emptied in second second

Or

minutes

Minutes.

Sol.13

Let P the required location of the pole from the gate B be m, i.e. BP m. Now the
difference of the distance of the pole from the two gates AP-BP (or BP-AP) 7 m.
Therefore, AP = m.

Now, AB 13 m, an since AB is a diameter

(By Pythagoras theorem)

Material downloaded from myCBSEguide.com. 10 / 32


i.e.

i.e.

i.e.

So, the distance of the pole from gate B satisfies the equation

So, it would be possible to place the pole if this equation has real roots. To see if this is so or
not, let us consider it’s discriminant. This discriminant is

So, the given quadratic equation has two real roots, and it is possible to erect the pole on the
boundary of the park.

Solving the quadratic equation by the quadratic formula, we get

Therefore, or

Since is the distance between the pole and the gate B, it must be positive. Therefore,
will have to be ignored. So .

Thus the pole has to be erected on the boundary of the park at a distance of 5 m from the
gate B and 12 m from the gate A

Sol.14 The numbers between 1 to 100 divisible by 6

96 ,

Or ,

Or

Material downloaded from myCBSEguide.com. 11 / 32


Or

Or

Or

Or

Sol.15

We have XY AC (Given)

So, BXY A and (Corresponding angles)

Therefore (AA Similarity Criterion)

So, (Theorem 6.6)

Also (Given)

So,

Material downloaded from myCBSEguide.com. 12 / 32


Therefore, from and

i.e. ,

or

or

or

or

Sol.16

Let the vertices of ABC is A(0,-1), B(2,1) and C(0,3)

D,E,F are mid of AB, BC & CA.

Midpoint formula

Co-ordinate of D

Co-ordinate of E

Co-ordinate of F

Vertices of DEF is D(1,0) , E(1,2) , F(0,1)

Here

Material downloaded from myCBSEguide.com. 13 / 32


Area of DEF

In ABC

Ratio

Or

Let the centre of the circumcircle be O(x, y). The points are A(-2, -3), B(-1, 0) and C(7, -6).

OA = OB = Radius

Also, OB = OC = Radius

Material downloaded from myCBSEguide.com. 14 / 32


adding (i) and (ii), we get
5x = 15
=> x = 3
put value of x in (i) we get,
=> 3y = -9
=> y = -3
So the circumcenter is (3,-3)

Sol.17

In fig. A and B represent points on the bank on opposite sides of the river, so that AB is the
width of the river. P is a point on the bridge at a height of 4m , i.e. DP = 4m. we are interested
to determine the width of the river, which is the length of the side AB of the APB.

Now, AB = AD + DB

In right APD,

So,

Material downloaded from myCBSEguide.com. 15 / 32


Also in right

So,

DB = 4m

AB = AD + DB

m
Therefore, the width of the river is m

Or

Let AN be the surface of the lake and O be the point of observation such that OA = h meters

Let P be the position of the cloud and P' be its reflection in the lake.
Then PN = P'N
Let

Material downloaded from myCBSEguide.com. 16 / 32


From (i) and (ii), we get

Sol.18

Let O be the centre of the concentric circles of radii 25 cm. and 7 cm. respectively. Let AB be
a chord of the larger circle touching the smaller circle at P. Then

Material downloaded from myCBSEguide.com. 17 / 32


AP = PB and OP AB

Applying Pythagoras theorem in OPA, We have

cm.

cm.

Sol.19

Steps of Construction:

(1) Draw a line segment AB = 8 cm.

(2) Taking A as centre draw a circle of radius = 4cm.

(3) Draw a perpendicular bisect of AB and let it intersect AB in P.

(4) With P as centre and PA or PB as radius, draw another circle intersecting the given circle
in Q & R.

(5) Join BQ and BR.

Thus BQ and BR are the required tangents from point P to the circle. Lengths BQ =BR = 6.9

Material downloaded from myCBSEguide.com. 18 / 32


cm.

Sol.20

Let A (8,6) , B (8, 2) and C ( 2, 2) be the vertices of given the triangle and Let P( ) be the
circumference of the triangle

Then PA = PB = PC

Taking

Again, taking

Material downloaded from myCBSEguide.com. 19 / 32


=> x = 5

Coordinates of P are (5, 2)

Radius PA

Sol.21

Pocket Money Number of Cumulative


(In Rupees) Employees (f) Frequency

5-10 3 3

10-15 6 9

15-20 9 18

20-25 11 29

25-30 14 43

30-35 10 53

35-40 7 60

This lies in class 25-30

Median group in 25-30

Now using formula

where

Material downloaded from myCBSEguide.com. 20 / 32


Rs.

Or

Since the maximum frequency = 8 and it corresponds to the class 3-5

Modal class = 3-5

Here,

We know that mode Mo is given by

Sol.22 There are 30 discs in the box out of which one disc can be drawn in 30 ways.

Total number of elementary events 30

(i) From number 1 to 30, there are 21 two digit numbers, namely 10,11,12….30 out of these 21
two digit numbered disc can be chosen in 21 ways.

Favourable number of elementary events 21

Hence P (getting a to digit numbered disc)

(ii) Those numbers from 1 to 30 which are perfect square 1,4,9,16,25 i.e. squares of 1, 2,3,4,5

Material downloaded from myCBSEguide.com. 21 / 32


respectively. Therefore, there are 5 disc marked with the number which are perfect squares.

Favourable number of elementary events 5

Hence, P (Getting a disc marked with a number which is perfect square)

Sol.23 Proof : Let us assume, to the contrary that is rational.

So, we can find integers r and such that

Suppose r and s have a common factor other than 1. Then we divide by the common factor to
get , where a and b are coprime.

So, we can write for some integer c.

Substituting for a we get , that is, .

This mean that 2 divides and so 2 divides b (again using theorem 1.3 with p =2 ).

Therefore, a and b have at least 2 as a common factor.

But this contradict the fact that a and b have no common factor other than 1.

This contradiction has arisen because of our incorrect assumption that is rational.

So, we conclude that is irrational.

Or

According to Euclid’s division lemma,


Given positive integers a and b, there exist unique integers q and r satisfying
a = bq + r, 0 ≤ r < b.
Let a be any positive integer and b = 3.
Then, let a = 3q + r, where q > 0 and 0< r < 3
a = 3q or a = 3q + 1 or a = 3q +2
We have three cases :

i) when a = 3q, then, a3 = (3q)3 = 9(3q3) = 9k [where k = 3q3 is an integer]


ii) when a = 3q + 1,

then, a3 = (3q + 1)3 = 9(3q3) + 9(3q2) + 9q + 1

Material downloaded from myCBSEguide.com. 22 / 32


= 9(3q3 +3q2 +3q) + 1 = 9k1 + 1 [where k1 = 3q3 + 3q2 + 3q is an integer]

iii) When a = 3q + 2

then, a3 = (3q + 2)3 = 9(3q3) + 9(6q2) + 9(4q) + 8 = 9(3q3 + 6q2 + 4q) + 8 = 9k2 + 8

[where k2 = 3q3 + 6q2 + 4q is an integer]


Therefore, the cube of any positive integer is of the form 9m, 9m + 1 or 9m + 8.

Sol.24 Dividend

and Divisor

So quotient

Remainder

Now Divisor quotient + remainder

Dividend

Material downloaded from myCBSEguide.com. 23 / 32


In this way, the division algorithm is verified.

Sol.25 Let the cost of one bat Rs.

And cost of one ball Rs.

Algebraically form

Solution by graphical method:

Table

x 200 100 0

y 50 100 150

Material downloaded from myCBSEguide.com. 24 / 32


x 225 100 75

y 25 75 125

Plot the points and draw the lines passing through them to represent the equations as shown
in figure.

The two lines intersect at point ( 150, 75)

So is the required solution of the pair of linear equation

Cost of one bat = Rs. 150 & Cost of one ball = Rs. 75

Cost of one bat & one ball 150+75 = Rs. 225

Or

Let the required numbers be x and y.

Then, x + y = 16

Material downloaded from myCBSEguide.com. 25 / 32


And,

xy = 48

We can write

x – y =

∴x + y = 16…(i)

x – y = 8…(ii)

Or, x + y = 16…(iii)

x – y = –8…(iv)

On solving (i) and (ii), we get x = 12 and y = 4

On solving (iii) and (iv), we get x = 4 and y = 12

Thus, the required numbers are 12 and 4.

Sol.26

Material downloaded from myCBSEguide.com. 26 / 32


BE and CF are medians of the ABC in which

From ABC,

(Pythagoras Theorem )

From ABE,

or (E is midpoint of AC )

or

or

From FAC,

or ( F is the mid-point of AB)

or

or

Adding and we have

Material downloaded from myCBSEguide.com. 27 / 32


i.e. [from (i)]

Sol.27 L.H.S.

R.H.S.

Sol.28

OE AB

BE AB

Material downloaded from myCBSEguide.com. 28 / 32


m

Area of segment

Area of two segment

Area of square lawn

Sum of the areas of the lawn and the flower beds

Ans.

Sol.29 Radius of cylinder m.

Height of cone m

and height of cylinder m

Material downloaded from myCBSEguide.com. 29 / 32


Capacity of the tent = Vol. of cone + Vol. of cylinder

Curved surface area of tent = Curved surface area of cone + Curved surface area of cylinder

Cost of canvas per Rs. 100

Cost of canvas 75.9 Ans.

Material downloaded from myCBSEguide.com. 30 / 32


Sol.30

Class - interval Frequency (f) c.f.

0 100 2 2

100 200 5 7

200-300 x 7 + x

300 - 400 12 19 + x

400 - 500 17 36 + x

500 - 600 20 56 + x

600 - 700 y 56 + x+ y

700 - 800 9 65 + x +y

800- 900 7 72 + x + y

900 - 1000 4 76 + x + y

Given n = 100

or

Median is 525 which lies interval 500-600

Median

525 = 500

Or

Or

Or

Material downloaded from myCBSEguide.com. 31 / 32


From equation

Or

Marks Cf Marks Cf

Less than 5 4 More than 0 100

Less than10 10 More than 5 96

Less than15 20 More than 10 90

Less than 20 30 More than 15 80

Less than 25 55 More than 20 70

Less than 30 77 More than 25 45

Less than 35 95 More than 30 23

Less than 40 100 More than 35 5

Material downloaded from myCBSEguide.com. 32 / 32


SAMPLE QUESTION PAPER 08
Class-X (2017–18)
Mathematics

Time allowed: 3 Hours Max. Marks: 80


General Instructions:
(i) All questions are compulsory.
(ii) The question paper consists of 30 questions divided into four sections A, B, C and D.
(iii) Section A contains 6 questions of 1 mark each. Section B contains 6 questions of 2 marks
each. Section C contains 10 questions of 3 marks each. Section D contains 8 questions of 4
marks each.
(iv) There is no overall choice. However, an internal choice has been provided in four
questions of 3 marks each and three questions of 4 marks each. You have to attempt only one
of the alternatives in all such questions.
(v) Use of calculators is not permitted.

SECTION – A

1. The sum and product of zeros of a quadratic polynomial are and –7 respectively.
Write the polynomial ?

2. Can two positive integers have their H.C.F and L.C.M as 12 and 512 respectively ? Justify.

3. If , then determine the value of .

4. Write the relation between Mean, Mode and Median.

5. If the straight line joining two points P (5, 8) and Q (8, k) is parallel to x-axis, then write the
value of k.

6. A tangent PQ at a point P of a circle of radius 5 cm meets a line through the centre O at a


point Q so that OQ = 12 cm. Write the length of PQ .

SECTION – B

Material downloaded from myCBSEguide.com. 1 / 21


7. The 7th term of an A.P. is – 4 and its 13th term is – 16. Find the sum of its first 19 terms.

8. If the points (4, 3) and (x, 5) lie on the circumference of the circle whose centre is (2, 3),
then find the value of x.

9. Show that is irrational.

10. In Fig-1, if EF||BC and FG||CD, prove that, .

11. A quadrilateral ABCD is drawn to circumscribe a circle (fig-2).


Prove that, AB + CD = AD + BC.

12. From a solid cylinder whose height is 2.4 cm and diameter 1.4 cm, a conical cavity of the
same height and same diameter is hollowed out (fig-3). Find the total surface area of the
remaining solid.

SECTION – C

13. Find the roots of the equation 3x2 – 7x – 2 = 0 by the method of completing the square.

Material downloaded from myCBSEguide.com. 2 / 21


14. Solve the pair of linear equations 8x + 5y = 9 and 3x + 2y = 4 by cross-.multiplication
method.

15. Poved that if in two triangles, sides of one triangle are in the same ratio of the sides of the
other triangle, then their corresponding angles are equal.

16. Prove that the points A(– 5, 4), B(–1, –2) and C(5, 2) are the vertices of an isosceles right-
angled triangle.

Or

The vertices of a triangle are A (-1, 3), B (1, -1) and C (5, 1). Find the length of the median
through the vertex C.

17. Cards marked with numbers 3, 4, 5, …, 50 are placed in a box and mixed thoroughly. One
card is drawn at random from the box. Find the probability that number on the drawn card
is a two digit number which is a perfect square.

Or

A die is thrown once. Find the probability of getting (i) an even number (ii) a number greater
than 3 (iii) a composiite number

18. A die is thrown once. Find the probability of getting (i) a prime number; (ii) an odd
number.

19. Solve for x :

Or

If the roots of the equation (b – c)x2 + (c – a)x + (a – b) = 0 are equal,


then prove that 2b = a + c.

20. If , then prove that,

21. Two tangents PA and PB are drawn to a circle with centre O from an external point P.
Prove that ∠ APB = 2 ∠ OAB.

Material downloaded from myCBSEguide.com. 3 / 21


Or

A circle with centre O, diameter AB and a chord AD is drawn. Another circle is drawn with
AO as diameter to cut AD at C. Prove that BD = 2OC.

22. State the ‘Fundamental Theorem of Arithmetic’ . Use Euclid’s division algorithm to find
the HCF of 196 and 38220. Hence find the LCM of these numbers.

SECTION – D

23. Draw a pair of tangents to a circle of radius 5 cm which are inclined to each other at an
angle of 60°.

Or

Draw a line segment AB of length 8 cm. Taking A as centre, draw a circle of radius 4 cm. and
constant the pair of tangents of the circle from point B and measure their lengths.

24. Check graphically whether the pair of equations x + y = 8 and x – 2y = 2 is consistent. If so,
solve them graphically. Also find the coordinates of the points where the two lines meet the
y-axis.

25. If and show that,

Or

If prove that

26. If the median of the distribution given below is 28.5, find the values of x and y.The sum of
all frequency is 60

Material downloaded from myCBSEguide.com. 4 / 21


Class-interval 0-10 10-20 20-30 30-40 40-50 50-60

Frequencry 5 x 20 15 y 5

27. Find the area of the shaded region in fig-5, where ABCD is a square of side 20cm.

28. The angle of elevation of a cloud from a point 60 m above a lake is 30o and the angle of

depression of the reflection of the cloud in the lake is 60o. Find the height of the cloud.

29. A metallic right circular cone 20 cm high and whose vertical angle is 60o is cut into two
parts at the middle of its height by a plane parallel to its base. Find the volume of the frustum
so obtained.

30. The term and the sum of first n terms of an A.P are respectively are and and
. Prove that, .

Or

Find the sum of first 40 positive integers divisible by 6. Also find the sum of first 20 positive
integers divisible by 5 or 7.

Material downloaded from myCBSEguide.com. 5 / 21


CBSE SAMPLE PAPER 08
CLASS X - Mathematics
Solutions

SECTION – A

1. Sum of zeroes =

Product of zeroes =

Coefficient of x2 = 5 and coefficient of x = 1 and the term free from x = – 35

The polynomial is

2. No. There can not exist two numbers satisfying the given condition, because here L.C.M (=
512) is not divisible by H.C.F ( = 12).

3. i.e.,

4. 3 Median = Mode + 2 Mean

5. k = 8

6.

SECTION – B

7. 7th term, . By question, a + 6d = – 4 … (i)

13th term, . By question, a + 12d = – 16 … (ii)

Material downloaded from myCBSEguide.com. 6 / 21


Now sum of first 19 terms

[ Adding (i) and (ii), we get, 2a+ 18d = – 16 ]

8. Length of radius =

i.e.,

i.e., (x – 2)2 = 0

i.e., x = 2

Hence, value of x = 2.

9. If possible, let us assume that is rational and equals to

i.e., , where a and b are positive integers prime to each other and b >1

i.e., ….. (i)

From (i), we see that, is not an integer, as a and b are prime to each other, so
are also prime to each other, but 3b is an integer

i.e in (i), a fraction equals to an integer, which contradicts our initial assumption.

Hence, is irrational. 1

10. Since, EF||BC,

Since, FG||CD,

Material downloaded from myCBSEguide.com. 7 / 21


By (i) and (ii),

11. Since, the lengths of tangents drawn from an external point to a circle are equal.

AP = AS … (i) BP = BQ … (ii)

CQ = CR … (iii) DR = DS … (iv)

Now, AB + CD

= AP + PB + CR + RD

= AS + BQ + CQ + DS

= (AS + DS) + (BQ + CQ)

= AD + BC

Hence proved.

12. Height of the solid cylinder (h)= 2.4 cm

Diameter of its base (2r)= 1.4 cm.

Material downloaded from myCBSEguide.com. 8 / 21


Therefore its base radius (r)= 0.7 cm

Height and diameter of the conical cavity are equal to those of the cylinder.

Remaining surface area

= (Curved surface area( outside) of cylinder) + (surface area of its bottom) + ( curved surface
area of the conical cavity)

= 1

SECTION – C

13.

i.e., [multiplying both sides by 3 ]

i.e.,

i.e.,

i.e.,

i.e.,

Material downloaded from myCBSEguide.com. 9 / 21


i.e.,

i.e.,

Therefore, the roots are and

14.

Solving equations (i) and (ii) by cross-multiplication method, we get,

i.e.,

i.e.,

i.e.,

Hence the solutions are x = – 2 , y = 5.

15. Let ABC and DEF be two triangles such that,

To prove that,

From AB and AC cutting AP = DE, AQ = DF let us join P and Q

Material downloaded from myCBSEguide.com. 10 / 21


so,

PQ || BC [If a line divides any two sides of a triangle in the same ratio, then the line is
parallel to the third side.]

So, (common angle)

Therefore, DABC and DDEF are equiangular and so their corresponding sides are in the same
ratio.

Hence, i.e.,

So

i.e., PQ = EF

DEF APQ (S-S-S)

So

Hence the corresponding angles of the triangles are equal.

16. Length of units,

Length of units,

Length of units,

Material downloaded from myCBSEguide.com. 11 / 21


Here we get,

Hence the triangle is an isosceles triangle.

Also, we observe that

Therefore, by Pythagoras theorem, DABC is a right-angled triangle (right angle at B)

Hence the triangle is an isosceles right-angled triangle.

17. According to the question, cards are mixed thoroughly and one card is drawn at random
from the box, so the event of drawing a card is equally and likely.

Since cards are marked with numbers 3, 4, 5, …, 50 ,

So there are 48 cards

Here the total number of possible outcomes = 48.

Let the event of drawing a card at random bearing two digit perfect square number be E.

Then the number of outcomes favourable to the event E = 4

(here two digit perfect numbers are 16, 25, 36, 49 )

Therefore,

18. When a die is thrown once, then the number of total outcomes = 6

Let the E be the event of getting one prime number.

Here prime numbers are 2, 3, 5

Then the number outcomes favourable to E = 3

Hence,

Again let F be the event of getting an odd number.

Here odd numbers are 1, 3, 5

Material downloaded from myCBSEguide.com. 12 / 21


Hence,

19. (x 1, 2, 3)

i.e., ( given x 1, 2, 3)

i.e.,

i.e.,

i.e.,

i.e., x = 0 or x = 4

Hence the solutions are x = 0 and x = 4.

20. Given,

i.e.,

i.e., [multiplying both sides by ]

i.e.,

i.e.,

i.e.,

Hence proved.

21. According to the question, from an outside point P two tangents PA and PB are drawn to a
circle with centre O (fig-4).

To prove that, ∠APB = 2 ∠ OAB.

Material downloaded from myCBSEguide.com. 13 / 21


Since the lengths of tangents drawn from an external point to a circle are equal.

So, PA = PB. i.e., PAB is isosceles.

[since, radius through point of contact is perpendicular to the tangent at the point of contact]

Hence proved.

22. Fundamental Theorem of Arithmetic :

Every composite number can be expressed ( factorised) as a product of primes, and this
factorisation is unique, apart from the order in which the prime factors occur.

Since 867 > 255, we apply the division lemma to 867 and 255, to get

867 = 255 3 + 102

255= 102 2 + 51

102= 51 2 + 0

The remainder is 0 (zero) and divisor is 51.

Therefore the HCF of 867 and 255 is 51.

Hence LCM of 867 and 255

SECTION – D

23. Construct a circle of radius 5 cm.

Let its centre be O.

Material downloaded from myCBSEguide.com. 14 / 21


Now construct one radius (say OP) of the circle.

At O, draw an angle of 60°,

and draw a perpendicular to OP at P, 90°.

Extend these lines to meet at T (say).

Now, taking T as centre and a radius equal to TP

draw an arc which cuts the circle at Q. Join T, Q.

Hence TP and TQ are the two tangents to the given circle of radius 5 cm

which are inclined to each other at an angle of 60°

[For justification of the construction:

i.e., ]

24.

x 0 4 8

y = 8 –x 8 4 0

Three solutions for equation (i)are given in the table :

Three solutions for equation (ii) are given in the table :

x 0 2 8

Material downloaded from myCBSEguide.com. 15 / 21


–1 0 3

Drawing Line AC

Drawing Line PR

Plotting points A(0, 8), B(4, 4) and C(8, 0) on graph paper the straight line AC is obtained as
graph of the equation
(i) Plotting points P(0, –1), Q(2, 0) and R(8, 3) on graph paper the straight line PR is obtained
as graph of the equation
(ii) From the graph, it is clear that a point M(6, 2) common to both the lines AC and PR.

So the pair of equations is consistent and the solutions of the equations are x = 6 and y = 2.

From the graph it is seen that the coordinates of the points where the lines AC and PR meets
the y-axis are (0, 8) and (0, –1) respectively.

25. Given, … (i)

and … (ii)

adding (i) and (ii), we get,

and subtracting (ii) from (i), we get,

Material downloaded from myCBSEguide.com. 16 / 21


. Hence proved.

26.

Class interval Frequency Cumulative Frequency

0-10 5 5

10-20 x 5+x

20-30 20 25+x

30-40 15 40+x

40-50 y 40+x+y

50-60 5 45+x+y

Total 60

It is given that, n = 60

i.e., 45 + x + y = 60 i.e., x + y = 15

The median is 28.5, which lies in the class 20-30

So, l = 20, f = 20, cf = 5+x, h = 10

We know,

Material downloaded from myCBSEguide.com. 17 / 21


Here,

Therefore y = 15 – 8 = 7.

27. Let the square be ABCD of side 20 cm.

Area of the square ABCD = .

Diameter of each circle (in fig-5) =

Therefore radius of each circle = 5 cm.

So area of each circle =

Total area of four squares =

Hence area of the shaded region in the fig-5

= 1

28. Let in the adjacent figure EC be the surface of water in the lake.

A is the position of the observer. AE = 60 m.

Also let B is the position of cloud and D be its image for fig.

Material downloaded from myCBSEguide.com. 18 / 21


In the lake and BF = h metre

So, BC = CD = (h + 60)m (see fig)

FC = 60 m.

By question,

In ABF,

In AFD,

[ since DF = DC + CF]

By (i) and (ii),

Hence height of the cloud from the water surface of the lake

=BC = 60 + 60 m = 120 m

29. Let ADH be a metallic right circular cone, whose height is 20 cm. is cut into two parts at

Material downloaded from myCBSEguide.com. 19 / 21


the middle of its height by a plane parallel to its base. The frustum is EBFHCDE

Given that, ,

AC = 20 cm 1

According to question, AB = BC = 10 cm.

In ABF,

Again in In ACH,

Therefore volume of the frustum EBFHCDE =

= [here H = AC = 20 cm AB = 10 cm]

Hence the required volume =

30. Let first term and the common difference of the AP be a and d respectively.

Then

Material downloaded from myCBSEguide.com. 20 / 21


According to question,

This is an identity.

Now putting, n = 2n – 1 and m = 2m– 1, we get,

Hence proved.

Material downloaded from myCBSEguide.com. 21 / 21


SAMPLE QUESTION PAPER 09
Class-X (2017–18)
Mathematics

Time allowed: 3 Hours Max. Marks: 80


General Instructions:
(i) All questions are compulsory.
(ii) The question paper consists of 30 questions divided into four sections A, B, C and D.
(iii) Section A contains 6 questions of 1 mark each. Section B contains 6 questions of 2 marks
each. Section C contains 10 questions of 3 marks each. Section D contains 8 questions of 4
marks each.
(iv) There is no overall choice. However, an internal choice has been provided in four
questions of 3 marks each and three questions of 4 marks each. You have to attempt only one
of the alternatives in all such questions.
(v) Use of calculators is not permitted.

SECTION – A

1. Find the zeroes of the quadratic polynomial .

2. A ladder of length H metre makes an angle of 60o with the ground when placed against a
wall. If the distance between the feet of the ladder and the wall be 5 m, then determine H.

3. Captain of a team tosses two different coins, one golden colour and other of silver colour
simultaneously. What is the probability that he gets atleast one head ?

4. Write the sum of the first 100 natural numbers.

5. Find the coordinate of the mid-point of the line segment joining the points whose

coordinates are and .

6. From an outside point A, two tangents AB and AC are drawn to touch the circle with centre

at O (in fig-1). Given BAC = 30o. Find AOB.

Material downloaded from myCBSEguide.com. 1 / 21


SECTION – B

7. Find the values of y for which the distance between the points P(2, – 3) and Q(10, y) is 10
units.

8. Find the discriminant of the equation and hence find the nature of
its roots.

9. The sum of first 30 and 40 terms of an AP are respectively 2265 and 4020, then find the
common difference of the AP.

10. The height and base diameter of a solid cylinder are 16 m and 2r m respectively. The
cylinder is melted and recast to 12 solid spheres of same base diameter. Find r.

11. Two concentric circles are of radius 7 cm and cm. Find the length of the chord of the
bigger circle, which touches the smaller circle.

12. If ABC and DEF are two similar triangles and their areas are 81 cm2 and 144 cm2
respectively. The bases of the triangles are respectively BC and EF. If EF = 15 cm, find BC.

SECTION – C

13. The denominator of a fraction is one more than thrice of the numerator. If the sum of the
fraction and its reciprocal is , then find the fraction.

14. Use Euclid’s division lemma to show that the cube of any positive integer is of the form
9m, 9m + 1 or 9m + 8.

15. An army contingent of 616 members is to march behind an army band of 32 members in

Material downloaded from myCBSEguide.com. 2 / 21


a parade. The two groups are to march in the same number of columns. What is the
maximum number of columns in which they can march?

16. The ratio of the sum of n-terms of two APs is (2n + 4) : (5n +2), find the ratio of their 12th
terms.

Or

Find the sum of all 3 digit numbers which leave remainder 3 when divided by 5.

17. The vertices of a ABC are A(4, 9), B and C(9, 4). A line is drawn to intersect

sides AB and AC at P and Q respectively, such that . Find the area of APQ.

Or

The vertices of a ABC is (1, 2), (3, 1) and (2, 5). Point D divides AB in the ratio 2:1 and P is
the mid-point of CD. Find the coordinates of the point P.

18. In fig-2, ABC and ABD are on the same base AB and on opposite sides of AB. If CD
intersects AB at O, then show that,

19. In fig-3, PQ and RS are two parallel tangents to a circle with centre O and another tangent
EF with point of contact C intersecting PQ at E and RS at F.
Prove that ∠EOF = 90°.

Material downloaded from myCBSEguide.com. 3 / 21


Or

Prove that, the lengths of tangents drawn from an external point to a circle are equal.

20. Prove that: .

21. A bag contains 5 red ball and some blue balls. If the probability of drawing a blue ball is
double that of a red ball, determine the number of blue balls in the bag.

Or

A bag contains 8 red balls and x blue balls. the odd against drawing a blue ball are 2:5. what
is the value of x?

22. One card is drawn from a well-shuffled deck of 52 cards. Find the probability of getting :
a) a king of red colour (b) a spade (c) a face card

SECTION – D

23. If we add 1 to the numerator and subtract 1 from the denominator, a fraction reduces to
1. It becomes if we only add 1 to the denominator. What is the fraction ?

Or

The sum of a 2 digit number and number obtained by reversing the order of the digits is 99.
If the digits of the number differ by 3. Find the number.

24. Find the common difference of an A.P. whose 1st term is 100 and the sum of whose first
six terms is 5 times the sum of the next six terms.

Material downloaded from myCBSEguide.com. 4 / 21


25. Draw a circle with the help of a bangle. Take a point outside the circle. Construct the pair
of tangents from this point to the circle.

Or

Let ABC be a right triangle in which AB = 6 cm, BC = 8 cm and ∠ B = 90°. BD is the


perpendicular from B on AC. The circle through B, C, D is drawn. Construct the tangents from
A to this circle.

26. Prove that,

27. Find the area of the segment APB shown in fig-4, if radius of the circle is 14 cm,
and (use )

28. There are two poles either on each bank of a river, just opposite to each other. One pole is
60 metre high. From the top of this pole, the angles of depression of the top and the foot of

the other pole are 30o and 45o respectively. Find the width of the river and the height of the
other pole.

29. In fig-5, a toy is in the form of a cone mounted on a hemisphere of common base of radius
7 cm. The total height of the toy is 31 cm. Find the total surface area of the toy. [ ]

Material downloaded from myCBSEguide.com. 5 / 21


30. The following frequency distribution gives the monthly consumption of electricity of 68
consumers of a locality. Find the median and mode of the data.

Monthly
65-85 85-105 105-125 125-145 145-165 165-185 185-205
Consumption

Number of
4 5 13 20 14 8 4
Consumer

Or

If the mean of the following distribution is 27, find the value of p. Also find the median and
mode.

Class-interval 0-10 10-20 20-30 30-40 40-50

No of workers 8 p 12 13 10

Material downloaded from myCBSEguide.com. 6 / 21


CBSE SAMPLE PAPER 09
CLASS X - Mathematics
Solutions

SECTION- A

1. 3x2 + 7x + 2

So the zeroes are .

2.

Therefore, H = 10 m.

3. Total outcomes = 22 = 4

4. Sum of the first 100 natural numbers =

5. Coordinates of the required mid-point = i.e.,

6. Since tangents drawn from an external point to a circle subtend equal angles at the centre
of the circle.

Material downloaded from myCBSEguide.com. 7 / 21


=

SECTION- B

7. By question,

i.e.,
i.e.,
i.e.,
i.e.,

Hence values of y are 3, – 9

8. Discriminant of is = 80 - 80 = 0

Therefore roots of the given quadratic equation are real and equal.

9. We know that the sum of first n-terms of an AP (whose first term is and common
difference is d) is

By question,

Again by question,

Subtracting (i) from (ii), we get,


10d = 50 i.e., d = 5
Hence the common difference of the AP is 5.

10. The volume of the solid cylinder =

Volume of each sphere =


[since its base diameter is equal to that of the cylinder]

By question, . Hence r = 1 m

Material downloaded from myCBSEguide.com. 8 / 21


11. Let C is the centre of two concentric circles of radii 5cm and 3 cm.

Let AB, a chord of the bigger circle which touches the smaller circle at M.
CM = cm, CA = 7 cm
[radius through point of contact is to tangent]
[by Pythagoras Theorem ]

i.e.,

Therefore, AB = 2AM = 2´6 = 12 cm.


[Line segment, drawn from centre of a circle perpendicular to any chord, bisects the chord ]
Hence, the length of the chord is 12 cm.

12. We know that, the ratio of the areas of two similar triangles is equal to the square of the
ratio of their corresponding sides.
Therefore for the given problem,
Hence, BC = 11.25 cm

SECTION – C

13. Let the fraction be


According to first condition, y = 3x + 1.
So the fraction becomes

Again by the 2nd condition,

Material downloaded from myCBSEguide.com. 9 / 21


Therefore. x = 5 (since, x >0)
Hence the required fraction =

14. According to Euclid’s division lemma,


Given positive integers a and b, there exist unique integers q and r satisfying
a = bq + r, 0 ≤ r < b.
Let a be any positive integer and b = 3.
Then, let a = 3q + r, where q > 0 and 0< r < 3
a = 3q or a = 3q + 1 or a = 3q +2
We have three cases :

i) when a = 3q, then, a3 = (3q)3 = 9(3q3) = 9k [where k = 3q3 is an integer]


ii) when a = 3q + 1,

then, a3 = (3q + 1)3 = 9(3q3) + 9(3q2) + 9q + 1

= 9(3q3 +3q2 +3q) + 1 = 9k1 + 1 [where k1 = 3q3 + 3q2 + 3q is an integer]

iii) When a = 3q + 2

then, a3 = (3q + 2)3 = 9(3q3) + 9(6q2) + 9(4q) + 8 = 9(3q3 + 6q2 + 4q) + 8 = 9k2 + 8

[where k2 = 3q3 + 6q2 + 4q is an integer]


Therefore, the cube of any positive integer is of the form 9m, 9m + 1 or 9m + 8.

15. To find the maximum number of columns in which members can march,
we have to calculate the HCF of 32 and 616.
Using Euclid’s algorithm,
616 = 32 19 + 8,
32 = 8 4 + 0
Therefore the HCF of 32 and 616 is 8
Hence they can march in 8 columns each.

16. Let, be the first terms of the two APs also let be their common
differences respectively.
According to question,

This is an identity.

Material downloaded from myCBSEguide.com. 10 / 21


Putting n = 23, we get,

i.e.,

i.e.,

i.e.,

i.e.,

Hence ratio of their 12th terms is 50 : 117.

17. Since P divides AB internally in the ratio 2 : 3,

So, abscissa i.e., x-coordinate of P =

and ordinate i.e., y-coordinate of P =

Therefore coordinates of P are (3, 5)

Since, Q divides AC internally in the ratio 2 : 3

Again, abscissa i.e., x-coordinate of Q =

and ordinate i.e., y-coordinate of Q =

Therefore coordinates of Q are (6, 7)

Using, formula : ,

area of APQ =

18. Given that, ABC and ABD are on the same base AB but on opposite sides of AB.

Material downloaded from myCBSEguide.com. 11 / 21


To prove that,

Construction :Let CM AB and DN AB are drawn.

Proof: In CMO and DNO,

CMO = DNO = 90o

COM = DON [ vertically opposite ]

CMO ~ DNO [by A-A property ]

So,

… (ii) and … (iii)

From (ii) and (iii) , Hence proved.

19. Given, two tangents PQ and RS, which touch the circle, with centre O, at points A and B.
Also, PQ || RS.

Another tangent EF touches the circle at C and meets the lines PQ and RS at E and F
respectively.

To prove, .

Material downloaded from myCBSEguide.com. 12 / 21


Construction : O, C joined.

Proof : .

[Since, radius through point of contact is perpendicular to the tangent]

In

OA = OC [radius of same circle]

EA = EC [Tangents drawn from outside point to a circle are equal]

i.e.,

Similarly, it can be shown that,

Since PQ || RC and EF is a transversal,

So,

. Hence proved.

20.

Material downloaded from myCBSEguide.com. 13 / 21


= RHS

Hence proved.

21. Let the number of blue balls in the bag be x.

Therefore total number of balls in the bag = 5 + x

Let probability of drawing a blue ball be P(blue).

Let probability of drawing a red ball be P(red).

Then,

and

According to the question, P(blue) = 2´ P(red)

i.e.,

i.e., x=10 [since x can not be negative ]

Hence number of blue balls in the bag is 10.

Material downloaded from myCBSEguide.com. 14 / 21


22. Total number of favourable outcomes = 52.

a) We know that, cards of diamond and heart are of red colour.

There exist one king of Diamond and one king of heart. Number of outcomes favourable to
‘getting a king of red colour’ = 2

P(getting a spade)

b) A well-shuffled deck of 52 cards contains 13 cards of spade.

So, number of outcomes favourable to ‘getting a spade’ = 13.

P(getting a king of red colour)

c) A well-shuffled deck of 52 cards contains 12 face cards.

So, number of outcomes favourable to ‘getting a face card’ = 12.

P(getting a face card)

SECTION - D

23. Let the numerator and the denominator be x and y respectively.

i.e., the fraction is

According to question we get, and

Material downloaded from myCBSEguide.com. 15 / 21


Now,

and

Substituting y in (ii) with the help of (i),

2x = x + 2 +1

i.e., x = 3 1

Now from (i), y = 3 +2 =5.

Hence the fraction is .

24. Let the 1st term and common difference of the AP be a and d respectively.

Here a = 100.

We know that sum of first n-terms of an AP, .

Now sum of first 6-terms of the AP is

and sum of next 6-terms of the AP

= S12 - S6

= 6(200 + 11d) - 3(200 + 5d)


= 1200 + 66d - 600 - 15d = 600 + 51d

According to the question,


600+15d = 5(600+51d)
i.e., 600 + 15d = 3000 + 255d
i.e., 600 – 3000= 255d –15d
i.e., 240d = – 2400
d = – 10
Hence the common difference of the AP is (– 10).

Material downloaded from myCBSEguide.com. 16 / 21


25.

Steps of construction :

Draw a circle with help of a bangle.


Take two non parallel chords AB and CD
Draw perpendicular bisectors of these chords.
Mark point O where two bisectors intersect. Point O is center of the circle.
Take a point P outside the circle and join PO.
Bisect PO. Let M be mid-point of PO.
Taking M as centre and MO as radius, draw a circle.
Let it intersect the given circle at the points Q and R.
Join PQ and PR.

26.

Hence proved.

27. Area of the segment APB = Area of the sector OAPBO – area of OAB.

Material downloaded from myCBSEguide.com. 17 / 21


Here area of the sector OAPBO
=

Let, OM AB is drawn. Given,

Since, perpendicular drawn from centre of a circle to a chord bisects the chord,

So, here,

Now

Therefore area of OAB =

Hence, Area of the segment APB

= Area of the sector OAPBO – area of OAB

28. Let two poles AB(= 60 m) and PQ be either on each bank of a river.

Material downloaded from myCBSEguide.com. 18 / 21


Let PQ = h m. and width of the river i.e., BQ = x m.

In figure, PR AB. AR = AB – BR = AB – PQ = (60 – h) m

Acc to question,

Angle of depression of P from A

= XAP = APR (alternate angle) =

and angle of depression of Q from A

= XAQ = AQB (alternate angle) =

Now in AQB,

i.e., i.e., x =60

Now in APR,

i.e.,

i.e., [since x = 60]

i.e.,

i.e.,

Material downloaded from myCBSEguide.com. 19 / 21


Hence the width of the river is 60 m and height of the other pole is 25.36 m.

29. Total height of the toy is 31 cm i.e., in fig-5, AD = 31 cm.

Radius of the common base is 7 cm.

i.e., OC = OB = OD = 7 cm.

AO = AB – OD = 31 – 7 = 24 cm.

Surface area of the toy

= slant surface area of the cone + curved surface area of the hemisphere

Here, slant surface area of the cone = sq. unit

Curved surface area of the hemisphere


=

Hence the total surface area of the toy =( 550 + 308) cm2 = 858 cm2.

30. For table = 1

Monthly consumption (in units) Number of consumer (fi) Cumulative frequency

65-85 4 4

85-105 5 9

105-125 13 22

Material downloaded from myCBSEguide.com. 20 / 21


125-145 20 42

145-165 14 56

165-185 8 64

185-205 4 68

Total N=68

For Median.

Here, , then , which lies in the interval 125-145.

Median class is 125-145.

Here, l = 125, n = 68, f = 20, cf = 22 and h = 20.

For Mode :

In the given data, maximum frequency is 20 and it corresponds to the class interval 125-145

Modal class = 125-145

and = 125, f1 = 20, f0 = 13, f2 = 14 and h = 20

Now,

Hence Median= 137 units, Mode = 135.77 units.

Material downloaded from myCBSEguide.com. 21 / 21


SAMPLE QUESTION PAPER 10
Class-X (2017–18)
Mathematics

Time allowed: 3 Hours Max. Marks: 80


General Instructions:
(i) All questions are compulsory.
(ii) The question paper consists of 30 questions divided into four sections A, B, C and D.
(iii) Section A contains 6 questions of 1 mark each. Section B contains 6 questions of 2 marks
each. Section C contains 10 questions of 3 marks each. Section D contains 8 questions of 4
marks each.
(iv) There is no overall choice. However, an internal choice has been provided in four
questions of 3 marks each and three questions of 4 marks each. You have to attempt only one
of the alternatives in all such questions.
(v) Use of calculators is not permitted.

SECTION - A

1. The decimal expansion of the rational number will terminate after how many places

of decimal?

2. Find the nature of the roots of the equation .

3. A boy walks 12m due east and 5 m due south. How far is he from the starting point?

4. If the point C(k, 4) divides the join of points A(2, 6) and B(5, 1) in the ratio 2 : 3, then find
the value of k.

5. Find the area of annulus whose inner and outer radii are 6 cm and 8 cm.

6. In a family of three children, find the probability of having at least one boy.

SECTION - B

7. Check whether on simplification gives a rational or irrational number.

Material downloaded from myCBSEguide.com. 1 / 21


8. Gunal saved Rs.10 in the first week of a month and then increased his weekly savings by
Rs.2.75. If in the nth week, his savings become Rs.59.50, find ‘n’.

9. Without using trigonometric tables prove that:


tan 1° tan 11° tan 21° tan 69° tan 79° tan 89° = 1

10. Find the third vertex of the triangle ABC. If two of its vertices are at A(-3, 1) and B(0, 2)

and the mid-point of BC is at D

11. D and E are points on the sides AB and AC respectively of a ABC. If AD = 5.7 cm, DB =
9.5 cm, AE = 4.8 cm and EC = 8 cm, then determine whether DE || BC or not.

12. Three cubes each of side 15 cm are joined end to end. Find the total surface area of the
resulting cuboid.

SECTION - C

13. Show that any positive odd integer is of the form (4m + 1 ) or ( 4m + 3), where m is some
integer.

14. If two zeroes of the polynomial x4 + 3x3 - 20x2 - 6x + 36 are and - , then find the
other zeroes of the polynomial.

15. Solve:

16. A class consists of a number of boys whose ages are in A.P., the common difference being
4 months. If the youngest boy is just eight years old and if sum of the ages is 168 years, then
find the number of boys in the class.

OR

Find for the AP in -9, -14, -19, -24…

17. Find the coordinates of the centroid of a triangle whose vertices are A( ),

OR

Material downloaded from myCBSEguide.com. 2 / 21


In figure, ABC is a right triangle right-angled at B. Medians AD and CE are of respective
lengths 5 cm and , find length of AC.

18. If then show that

19. Prove that:

OR

If , show that

20. A square field and an equilateral triangular park have equal perimeters. If the cost of

ploughing the field at rate of Rs 5 perm2 is Rs 720, find the cost of maintaining the park at

therate of Rs 10 per m2.

21. The following table shows the marks obtained by 100 students of class X in a school
during a particular academic session. Find the mode of this distribution.

Less Less Less Less Less Less Less Less


Marks
than 10 than 20 than 30 than 40 than 50 than 60 than70 than 80

No of
7 21 34 46 66 77 92 100
students

OR

The percentage of marks obtained by 100 students in an examination are given below:

Marks 30-35 35-40 40-45 45-50 50-55 55-60 60-65

Frequency 14 16 18 23 18 8 3

Material downloaded from myCBSEguide.com. 3 / 21


Determine the median percentage of marks.

22. A bag contains 6 red balls and some blue balls. If the probability of drawing a blue ball
from the bag is double that of a red ball, find the number of blue balls in the bag.

SECTION - D

23. Solve the following system of linear equations graphically: 3x + y = 12 and x - 3y = -6.
Shade the region bounded by these lines and the x-axis. Also find the ratio of areas of
triangles formed by given lines with x-axis and the y-axis.

OR

A boat goes 30 km upstream and 44 km downstream in 10 hours. In 13 hours it can go 40 km


upstream and 55 km down stream. Determine the speed of the stream and that of the boat in
still water.

24. A plane left 30 minutes late than its scheduled time and in order to reach the destination
1500 km away in time, it had to increase the speed by 250 km/h from the usual speed. Find its
usual speed.

25. Prove that the ratio of areas of two similar triangles is equal to the square of the ratio of
their corresponding sides. Use the above theorem, in the following. The areas of two similar

triangles are 81 cm2 and 144 cm2. If the largest side of the smaller triangle is 27 cm, find the
largest side of the larger triangle.

OR

In a triangle if the square of one side is equal to the sum of the squares on the other two
sides. Prove that the angle apposite to the first side is a right angle. Use the above theorem to
find the measure of in figure given below.

Material downloaded from myCBSEguide.com. 4 / 21


26. Prove that the intercept of a tangent between two parallel tangents to a circle subtends a
right angle at the centre of the circle.

27. Construct a triangle similar to given ABC in which AB = 4 cm, BC = 6 cm and ABC = 60°,
such that each side of the new triangle is of given ABC.

28. The angle of elevation of the top a tower at a point on the level ground is 30°. After
walking a distance of 100m towards the foot of the tower along the horizontal line through
the foot of the tower on the same level ground, the angle of elevation of the top of the tower
is 60°. Find the height of the tower.

29. A container (open at the top) made up of a metal sheet is in the form of a frustum of a
cone of height 16 cm with radii of its lower and upper ends as 8 cm and 20 cm respectively.
Find
(i) the cost of milk when it is completely filled with milk at the rate of Rs 15 per litre.

(ii) the cost of metal sheet used, if it costs Rs 5 per 100 cm2

30. The median of the following data is 20.75. Find the missing frequencies x and y, if the
total frequency is 100.

Class-Interval 0-5 5-10 10-15 15-20 20-25 25-30 30-35 35-40

Frequency 7 10 x 13 y 10 14 9

OR

In the following distribution, locate the median mean and mode.

Monthly consumption
65-85 85-105 105-125 125-145 145-165 165-185 185-205
of electricity

No. of Consumers 4 5 13 20 14 7 4

Material downloaded from myCBSEguide.com. 5 / 21


SAMPLE PAPER 10
(CLASS X - MATHEMATICS )
Marking Scheme

1.

The decimal expansion of will terminate after 4 places of decimal

2.

D = 108 > 0
Therefore the roots are real and distinct.

Let the starting point be B.


Then BC =12 m & CA = 5m
By using Pythagoras theorem,

The distance between the starting point and end point = 13 m.

Material downloaded from myCBSEguide.com. 6 / 21


By using section formula,

5. Area of Annulus =

6 Sample Space = { BBB, BBG, BGB, BGG , GBB, GBG, GGB,GGG}


n(S) = 8
P(having at least one boy) =

Section – B

Question numbers 7 to 12 carry two marks each

7.

= ( a rational number)

8. a = 10 ; d = 2.75
an = 59.50

59.50 = 10 + (n – 1) (2.75)
n = 19

OR

a = -9
d = -14 - (-9) = -14 + 9 = -5
a30 - a20 = (a + 29d) - (a + 19d)

Material downloaded from myCBSEguide.com. 7 / 21


= 10d = 10 x (-5) = -50

9. LHS = tan (90°-89°) tan (90°- 79°) tan (90°- 69°) tan 69° tan 79° tan 89°
= cot 89° cot 79° cot 69° tan 69° tan 79° tan 89°
= cot 89° cot 79° cot 69° × × ×

= 1

10. Let the third vertex be (x, y)

11. AD = 5.7 cm, DB = 9.5 cm, AE = 4.8 cm and EC = 8 cm


Since D and E are the points on AB and AC respectively.
Check whether

or not.

0.6 = 0.6

Therefore,
(each equal to 0.6)
Hence, by the converse of Thales theorem DE || BC

12. Resulting solid is a cuboid with length = 3(15) = 45 cm ; breadth = 15 cm ;


Height = 15 cm
TSA of the cuboid = 2 (lb + bh + hl)
= 2 ( 675 + 225 +675)

= 3150 cm2

Section C

Question numbers 13 to 22 carry three marks each

13. Let n be any arbitrary positive odd integer.


On dividing n by 4, let m be the Quotient and r be the remainder. So, by Euclid’s division
lemma, we have

Material downloaded from myCBSEguide.com. 8 / 21


n = 4m + r, where m ≠ 0 and r < 4.
As m ≠ 0 and r < 4 and r is an integer, r can take values 0, 1, 2, 3.
n = 4m or n = 4m + 1 or n = 4m + 2 or n = 4m + 3
n = 4m + 1 or n = 4m + 3( since n is odd)
Thus, any positive odd integer is of the form (4m + 1) or (4m + 3), where m is some integer.

14. Since and - are two zeroes of the polynomial

(x - ) (x + ) is a factor of the polynomial.

By long division method x4 + 3x3 - 20x2 - 6x + 36

= (x2 - 2) (x2 + 3x - 18)

= (x2 - 2) (x + 6) (x - 3)
The other zeroes of the Polynomial are -6,3. 1

15.

(x2- 5x + 4) +(x2 - 5x + 6)=

(x2 - 6x + 8)

6x2 – 30x +30 = 10x2 -60x +80

4x2 – 30x +50=0

2x2 – 15x +25 = 0


(x-5)(2x-5) = 0
x=5; x=5/2

16. 96,100, 104,.........


a = 96 , Sn = 168 ×12 = 2016 ; d = 4

Sn = (2×96 + (n – 1) 4)=2016

n2+47n-1008=0
(n+63)(n-16) =0
n=-63 is rejected; n =16
Therefore no of boys in the class = 16

17. The vertices of triangle are A(2 , ),

Material downloaded from myCBSEguide.com. 9 / 21


Therefore the coordinates of centroid are

18.

19

= RHS

OR

L.H.S

Material downloaded from myCBSEguide.com. 10 / 21


20. Let the side of the park be ‘a’ meter.

5 x a2 = 720 a = 12m.
Perimeter of square = 48 m.
Perimeter of triangle = 48m.
Side of triangle = 16m.
Now Area of triangle

= = 64 m 2.
Cost of maintaining the park
= Rs. (10 x 64 )
= Rs. 1108.48

21

No.of
Marks
students(fi)

0 – 10 7

10 - 20 14

20 - 30 13

30 - 40 12(f0)

40 - 50 20(f1)

50 - 60 11(f2)

60 - 70 15

70 - 80 8

TOTAL 68

Modal class is 40 – 50 because it has highest frequency

Mode =

Material downloaded from myCBSEguide.com. 11 / 21


OR

Marks (class) No. of students (Frequency) Cumulative Frequency

30-35 14 14

35-40 16 30

40-45 18 48

45-50 23 71

50-55 18 89

55-60 8 97

60-65 3 100

Here n = 100

Therefore , which lies in the class 45-50


l1 (The lower limit of the median class) = 45

c (The cumulative frequency of the class preceding the median class) = 48


f (The frequency of the Median class)= 23
h (The class size) = 5

Median

So, the median percentage of marks is 45.4

22. Let the number of blue balls be ‘x’.


Thus the sum of the possible outcomes = 6 +x

Material downloaded from myCBSEguide.com. 12 / 21


Now the sum of the favourable outcomes if the red balls are drawn = 6
P(E) =
Again the sum of the favourable outcomes if the blue balls are drawn = x
P(E) =
=2 ×

x=12
Hence the number of blue balls = 12.

Section – D

23

Since the lines intersect at (3, 3), there is a unique solution given by x=3, y = 3
Area of triangle ABC formed by lines with x - axis = ½ x 10 x 3 = 15sq. units
Area of triangle BDE formed by lines with y - a x is = ½ x 10 x 3 = 15 sq units
Ratio of these areas = 1 :1

OR

Material downloaded from myCBSEguide.com. 13 / 21


Let the speed of boat is }x\text{ km/h in still water
and stream y km/h
According to question,

on solving eq. (i) and (ii) we get,

on solving eq. (iii) and (iv) we get,


x = 8km/h
y = 3km/h

24. Let the usual speed of plane be x km/hour

Time taken = hrs. with usual speed

Time taken after increasing speed = hrs

Given that

x2 + 250x - 750000 = 0
(x + 1000 ) ( x -750 ) = 0
x = 750 or -1000 (Rejected)
Usual speed of plane = 750km/h.

25 Given: ABC ~ DEF


To Prove:

Construction: Draw AG BC and DH EF


Proof :

Material downloaded from myCBSEguide.com. 14 / 21


Now in ABG and DEH, B = E
AGB = DHE(Each 90°)
AGB ~ DHE

But,

From (i) and (ii), we get,

Similarly, we can prove

Let the largest side of the larger triangle be x cm, then


(Using the theorem)

x = 36 cm

OR

Given: A such that

To prove: Triangle ABC is right angled at B


Construction: Construct a triangle DEF such that

Material downloaded from myCBSEguide.com. 15 / 21


and
Proof: is a right angledtriangle right angled at E [construction]
By Pythagoras theorem, we have

Thus, in and we have


[By Construction and (i)]

Hence, is a right triangle.


In

Now in
is right angled at K

26

Material downloaded from myCBSEguide.com. 16 / 21


Since tangent is perpendicular to the radius of the circle
SPO = SRO = OQT = 90°
In right triangles OPS and ORS
OS = OS (common)
OP = OR (radii of circle)
OPS ORS(RHS congruence)
1 = 2
Similarly 3= 4
Now 1 + 2 + 3 + 4 =180°
2 + 3 =90°
SOT = 90°

27. DABC in which AB = 4 cm, BC = 6 cm and ABC = 60°


DA’BC’ is the required similar triangle.

Material downloaded from myCBSEguide.com. 17 / 21


28

In right BAC ,

AB = (100 + AD) x ---(i)

In right DBAD ,

AB = AD x ---(ii)
From (i) and (ii) we get
100 + AD = 3 AD
AD = 50 m
From (ii) AB = 50 m
= 50 x 1.732m
or, AB = 86.6 m

29. The Container is a frustum of cone h = 16cm, r = 8cm, R = 20cm

Volume of the container = x ph ( R2 + Rr + r2 )

= x 3.14 x 16 ((20)2 + 20(8) + (8)2) cm3

= x 3.14 x 16 (400 + 160 +64) cm3

= ( x 3.14 x 16 x 624 ) cm3

= (3.14 x 3328) cm3

= 10449.92 cm3
= 10.45 litres
Cost of milk = Rs (10.45 x 15) = Rs 156.75

Material downloaded from myCBSEguide.com. 18 / 21


Now, slant height of the frustum of cone = l

Total surface area of the container = pl ( R+r) +p r2

= 3.14 x 20 (20 + 8) + 3.14 (8)2 cm2

= 3.14 [ 20 x 28 + 64 ] cm2

= 3.14 x 624 = 1959.36 cm2


Cost of metal Used = Rs 1959.36 x
= Rs 19.5936 x 5
= Rs 97.968
= Rs 98 (Approx.)

30

CLASS INTERVAL FREQUENCY CUMULATIVE FREQUENCY

0-5 7 7

5-10 10 17

10-15 x 17+x

15-20 13 30+x

20-25 y 30+x+y

25-30 10 40+x+y

30-35 14 54+x+y

35-40 9 63+x+y

Given n(total frequency ) = 100


100 = 63 + x + y
x + y = 37 ----------(1)

Material downloaded from myCBSEguide.com. 19 / 21


The median is 20.75 which lies in the class 20-25
So, median class is 20-25
l = 20
f = y
c.f = 30 + x
h = 5
Using formula,

Median = l +

20.75 = 20 +

3y = 400 - 20x
20x + 3y = 400 ------(2)
Solving (1) and (2), we get
x = 17
y = 20

OR

Monthly consumption of electricity No. of consumers (F) C.F Class Mark (X) FX

65-85 4 4 75 300

85-105 5 9 95 475

105-125 13 22 115 1495

125-145 20 42 135 2700

145-165 14 56 155 2670

165-185 8 64 175 1400

185-205 4 68 195 780

∑ƒx=9320

Now and this is in 125-145 class


Median class = 125-145

Material downloaded from myCBSEguide.com. 20 / 21


Here,

We know that

Hence, Median = 137

Again Mean

For mode, since the maximum frequency is 20 and this corresponds to the class 125-145
Here,

Thus, Median = 137, Mean = 137.05 and Mode = 135.76


The three measures are approximately the same in the class.

Material downloaded from myCBSEguide.com. 21 / 21


SAMPLE QUESTION PAPER 11
Class-X (2017–18)
Mathematics

GENERAL INSTRUCTIONS

(i) All questions are compulsory.


(ii) The question paper consists of 30 questions divided into four sections A, B,C and D.
(iii) Section A contains 6 questions of 1 mark each, Section B contains 6 questions of 2 marks
each, Section C contains 10 questions of 3 marks each and Section D contains 8 questions of 4
marks each.
(iv) Use of Calculator and log tables is not permitted.

Q.1 Find the value of k for which the pair of linear equations and
has no solution

Q.2. for what value of p, are 2p + 1, 13, 5p-3 three consecutive terms of an A.P?

Q.3 without using trigonometry table, evaluate

Q.4 In figure, , BC = 7.5 cm , AM = 4cm and MC = 2 cm. Find the length BN

Q.5 The two tangents from an external point P to a circle with centre O and PA and PB. If
what is the value of ?

Q.6 A pair of dice is thrown once. Find the probability of getting the same number on each
dice.

Q. 7 Prove that is an irrational number.

Material downloaded from myCBSEguide.com. 1 / 27


Q.8 Solve the equations graphically:

What is the area of the triangle formed by the two lines and the line y = 0 ?

Q.9 Find the roots of the following equation

Q.10 Prove that :

Q.11 The angle of elevation of the top of a building from the foot of a tower is and angle
of elevation of top of the tower from the foot of the building is . If the tower is 50 m high.
Find the height of the building.

Q.12 The dimensions of a metallic cuboid are 100 cm 80 cm 64 cm. It is melted and
recast into a cube . Find the surface area of the cube.

Q.13 A person on tour has Rs. 4200 for his expense. If he extends his tour for 3 days , he has
to cut down his daily expense by RS. 70. Find the duration of the tour.

Q.14 The first and last term of an AP are 8 and 350 respectively. If its common difference is 9,
how many terms are there and what is their sum?

Q.15 Prove that the ratio of areas of two similar triangles is equal to the ratio of the square of
their corresponding sides.

Q.16 Point P divides the line segment joining the points A (2,1) and B (5, -8) such that
If P lies on the line , find the value of k.

OR

Show that the points and are the vertices of a square.

Q.17 From the top of a 7m high building, the angle of elevation of the top of a tower is
and angle of depression of the foot of the tower is Find the height of the tower.

OR

A girl who is 1.2 m tall, spots a balloon moving with the wind in a horizontal line at a height
of 88.2 m from the ground. The angle of elevation of the balloon from the eye of the girl at

Material downloaded from myCBSEguide.com. 2 / 27


any instant is . After sometime, the angle of elevation reduces to . Find the distance
travelled by the balloon during the interval.

Q.18 In figure, a triangle ABC is drawn to circumscribe a circle of radius 2 cm such that the
segment BD and DC into which BC is divided by the point of contact D are the lengths 4cm
and 3cm respectively. If area of ABC = 21 , then find the lengths of sides AB and AC.

Q.19 Draw a triangle ABC with BC = 7cm, and . then construct another
triangle whose sides are times the corresponding sides of ABC.

OR

Draw a circle of radius 2.3 cm take a point P on it. Without using the centre of the circle.
Draw tangent to it at P.

Q.20 The mid points of sides of a triangle are (3,4) , (4,1) and (2,0). Find the coordinate of
vertices of the triangle.

Q.21 Weekly income of 600 families is given below.

Income in Rs. Frequency

0 - 10000 250

1000 2000 190

2000 3000 100

3000 4000 40

4000 5000 15

Material downloaded from myCBSEguide.com. 3 / 27


5000 6000 5

Find the median.

OR

The following tables gives production yield per hectare of wheat of 100 farms of village:

Production Yeild (in hr.) No. of Farms

50 - 55 2

55 - 60 8

60 - 65 12

65 - 70 24

70 - 75 38

75 - 80 16

Change the distribution to a more than type distribution and draw its Ogive.

Q.22 All kings, queens and aces are removed from a pack of 52 cards. The remaining cards
are well shuffled and then a card id drawn from it. Find the probability that the drawn card
is
(i) a black face card
(ii) a red card.

Q. 23 Show that only one of the number n, n+2 and n+4 is divisible by 3.

OR

Use Euclid Division Lemma to show that cube of any positive integer is either of the form 9m,
(9m + 1) or (9m + 8).

Q. 24 Obtain all other zeroes of the polynomial , if two of its zeroes


are and .

Material downloaded from myCBSEguide.com. 4 / 27


Q.25 Solve the following pair of equations for and

OR

If in a rectangle the length is increased and breadth is decreased by 2 units each, the area is
reduced by 28 square units, and if the length is reduced by 1 unit and breadth is increased by
2 units, the area increased by 33 square units. Find the dimensions of the rectangle.

Q 26. In figure AB units, CD units and units, prove


that

Q.27 Prove that

Q.28 ABC is a right triangle, right angled at A. Find the area of shaded region if AB = 6cm, BC
= 10cm and O is the centre of incircle of ABC. (take )

Q.29 A gulab jamun , when ready for eating, contains sugar syrup of about 30% of its volume.
Find approximately how much syrup would be found in 45 such gulab jamuns, each shaped

Material downloaded from myCBSEguide.com. 5 / 27


like a cylinder with two hemispherical ends, if the complete length of each of them is 5cm
and its diameter is 2.8 cm.

Q.30 Draw ‘less than Ogive’ and ‘more than Ogive’ for the following distribution and hence
find its median.

Class Frequency

20 - 30 10

30 - 40 8

40 - 30 12

50 - 60 24

60 - 70 6

70 - 60 25

80 - 90 15

OR

Find the mean, mode and median for the following data.

Classes Frequency

5 - 15 2

15 - 25 3

25 - 35 5

35 - 45 7

45 - 55 4

55 - 65 2

65 - 75 2

Material downloaded from myCBSEguide.com. 6 / 27


SAMPLE PAPER 11
(CLASS X - MATHEMATICS )
Marking Scheme

1. Since pair of equations has no solution


Then,

i.e.

2. 2p+1, 13 ,5p-3 are consecutive terms of an A.P

13 - 2p - 1 = 5p - 16
12 - 2p = 5p - 16
12 + 16 = 5p + 2p
28 = 7p p = 4

3.

4. In MN AB

[ Let x = BN]

3x = 15 x = 5
Hence BN = 5 cm.

5. PA and PB are tangents to the circle

Material downloaded from myCBSEguide.com. 7 / 27


[ Tangent makes an angle of with the radius at the point of contact ]
In quadrilateral OAPB,

[Angle sum property of a quadrilateral]

6. Total number of outcomes when a pair of dice is thrown is 36


Same number on each dice i.e. (1,1) , (2,2) , (3,3) , (4,4) , (5,5) , (6,6)
Number of ways of getting the same number on each dice = 6.
Required probability =

7. Let if possible is a rational number.

From we notice
LHS is an irrational number and RHS is rational number, which is not possible. Hence, our
supposition is wrong. Hence, is an irrational number.

8.

Material downloaded from myCBSEguide.com. 8 / 27


1 0 2

0 2 -2

From

0 -4 2

2 0 3

From

From graph, we observe that solution of equation is (0,2)


Area
= 5 square units

9.

Material downloaded from myCBSEguide.com. 9 / 27


x2 - 3x - 28 = -30

x2 - 3x + 2 = 0
(x - 2)(x - 1)=0
x = 2,1
Required roots are 2, 1

10. Taking LHS

=RHS

11. Solution :

Let TP be height of tower of 50m


,

Material downloaded from myCBSEguide.com. 10 / 27


Let distance AP between building and tower be .
To find : AB, height of building
In right
In right

AB

AB = 16.67m
Hence height of the building is 16.67 m

12. Dimensions of the metallic cuboid are


100 cm 80 cm 64 cm
100 80 64
(Where a is side of cube)

a = 80 cm
Surface area of cube

13. Number of days for a tour = x


Daily expense = y
A.T.Q.

If tour be extended for 3 days


New number of days = x + 3
Daily expense = y - 70
Now (x + 3)(y - 70) = 4200
xy - 70x + 3y -210 = 4200
-70x + 3y - 210 = 0 [From (i)]
Put

Material downloaded from myCBSEguide.com. 11 / 27


x2 - 180 + 3x = 0

x2 + 3x - 180 = 0
(x + 15)(x - 12) = 0
x = 12, x = -15 (rejected)
Duration of Tours = 12 Hours

14. Here a = 8, l =350, d=9


From formula, l= an = a + (n - 1) d, we get

a + ( n-1) d=350
8 + (n - 1)9 = 350
(n - 1)9 = 350 - 8
(n - 1)9 = 342
n - 1 =
n - 1 = 38
n = 38 + 1

From formula, we get

15. Given :

To prove :
Construction : Draw AM BC and DK EF
Proof : ABC

Material downloaded from myCBSEguide.com. 12 / 27


Also ABM and DEK

(given) ( angles of similar s )


(Construction)
By AA Rule

Equating (i) and (ii)

So, Hence proved

16. P is the point of intersection of line segment AB and line 2x - y + k = 0.


Here,
3 AP = AP + PB
2 AP = PB
AP : PB = 1 : 2
P divides the line segment joining A (2,1) and B(5, -8) in ratio 1:2

Coordinates of point P are

i.e. p(3, -2)

As point P lies on the line 2x - y + k = 0.


6 + 2 + k = 0 6 = -8

OR

Material downloaded from myCBSEguide.com. 13 / 27


Diagonal

Diagonal
Hence proved.

17. AB is a building of height 7 m and CD is tower of height h m.

AB = ED = 7m
and CE = (h-7)m
Let BD = AE = x m
In right AED,

In right AEC,

Height of the tower is 28 m.

OR

In right

Material downloaded from myCBSEguide.com. 14 / 27


In right

18. Let AE = AF = x
Length of tangents from an external point is equal.

ar BOC =
ar AOB =
ar AOC =

Material downloaded from myCBSEguide.com. 15 / 27


[from (i)]

21x + 7x - 49 = 0

2x2 + 7x - 49 = 0

2x2 + 14x - 7x -49 = 0


2x(x + 7) - 7(x + 7) = 0
(2x - 7)(x + 7) = 0
[rejected]
length of side AB = 4+ 3.5 = 7.5 cm and AC =3 + 3.5 = 6.5 cm

19.

OR

Material downloaded from myCBSEguide.com. 16 / 27


Steps of construction:

1. Draw a circle of radius 2.3 cm and take a point P on it.


2. Draw chord PQ.
3. Mark a point R in the major arc QP.
4. Join PR and RQ.
5. Draw
6. Produce to as shown in figure, then is the required tangent at the point P.

20. Let A(x1 , y1), B(x2, y2) and C(x3 , y3) are the vertices of ABC.

(3,4), (4,1),(2,0) are mid points of sides AB,BC,CA


As (3,4) is mid point of AB
x1 + x2 = 6 ....(i)

y1 + y2 = 8 ...(ii)

As (4,1) are mid points of BC


x2 + x3 = 4 ...(iii)

y2 + y3 = 2... (iv)
As (2,0) are mid points of AC
x1 + x3 = 4 ...(v)

y1 + y3 = 0 ..(vi)

Adding (i), (iii), (v), we get


2 (x1 + x2 + x3) = 18

x1 + x2 + x3 = 9

as x1 + x2 = 6 [from (i)]

x3 = 3

x1 = 1, x2 = 5

Material downloaded from myCBSEguide.com. 17 / 27


Similarly, y1 = 3, y2 = 5, y3 = -3

Coordinate of vertices of are A(1, 3), B(5, 5), C(3, -3)

21.

Income in Rs. Number of families (f) c. f

0 - 1000 250 250

1000 - 2000 190 440

2000 - 3000 100 540

3000 - 4000 40 580

4000 - 5000 15 595

5000 - 6000 5 600

n 600 = 300
Median class = 1000 - 2000
l = 1000, c.f.=250, f=190, h=1000

Median

OR

More than type Ogive

Production yield (Kg/ha) C.F

More than or equal to 50 100

More than or equal to 55 98

More than or equal to 60 90

More than or equal to 65 78

Material downloaded from myCBSEguide.com. 18 / 27


More than or equal to 70 54

More than or equal to 75 16

Now, draw the Ogive by plotting the points (50, 100), (55, 98), (60, 90), (65, 78), (70, 54), (75, 16)

22. Number of kings = 4, number of queens =4,

number of aces = 4
After removing all kings, queens and aces,
number of cards left = 52 – 12 = 40

(i) Number of black face cards in the remaining cards ( 2 jacks) = 2


Probability of black face card

(ii) Out of 12 cards removed, 6 are of red colour.


Number of red- coloured cards left 26 – 6 = 20
Number of ways of drawing a red card = 20
Probability of getting a red card

23. Let n = 3k ; 3k+1 or 3k +2


(i) When n = 3k n is divisible by 3
n+ 2 = 3k +2
n + 2 is not divisible by 3
and n + 4 = 3k + 4 = 3k + 3 + 1
= 3(k +1) + 1
n + 4 is not divisible by 3.

Material downloaded from myCBSEguide.com. 19 / 27


(ii) When n = 3k + 1
n is not divisible by 3
n + 2 = (3k + 1) + 2 = 3k + 3
= 3 (k + 1)
n + 2 is divisible by 3
n + 4 = 3k + 1 + 4 = 3k + 5
= 3k + 3 + 2 = 3(k +1)+2
∴ n + 4 is not divisible by 3

(iii) When n = 3k + 2, n is not divisible by 3


n + 2 = (3k +2) +2 = 3k + 4
= 3(k +1) +1
n + 2 is not divisible by 3
n + 4 = (3k +2) + 4
= 3k + 6 = 3(k+2)
n + 4 is divisible by 3.
Only one of the numbers n , n + 2 and n + 4 is divisible by 3.

OR

Let a = 3q + r

a = 3q; then a3 = 27q3=9m ; where m=3q3

when a=3q+1 ; then a3 =27q3+27q2+9q+1

=9 (3q3+3q2+q)+1 = 9m+8 (where m = 3q3 + 3q2+q)

when a=3q+2 ; then a3= (3q+2)2

= 27q3+54q2+36q+8

= 9(3q3+6q2+4q)+8

= 9m+8 (where m = 3q3+6q2+4q)


Hence, cubes of any positive integer is either of the form9m,( 9m+1 ) or( 9m+8 ).

24. Solution :

are the zeroes of

Material downloaded from myCBSEguide.com. 20 / 27


is a factor of

is factor of

For other factor

other factor of p(x) is x2 - 3x + 2=0

For other zeroes, x2 - 3x + 2=0


(x - 2)(x - 1)
x = 2, x = 1
Other zeroes are 1 and 2.

25.
Put and

also ; put and

We have

Equation (i) - 2 x equation (ii)

_____________

Material downloaded from myCBSEguide.com. 21 / 27


x - y = 5 ..(iii)
Put

Solving (iii) and (iv) for x and y


We have

OR

Let the length and breadth of a rectangle be x and y meters.


According to question,
Area = xy
(x + 2) (y - 2) = xy - 28
or 2x - 2y = 24
or x - y = 12 ...(i)
and (x - 1) (y + 2) = xy + 33
2x - y = 33 ....(ii)
on subtracting eq (ii) - (i), we get,
x = 21 and 21 - y = 12
so y = 21 - 12
y = 9
length =21m, breadth = 9m

26.

Let BQ = units, DQ b units

Material downloaded from myCBSEguide.com. 22 / 27


and

Similarly

Also

From (i) and (ii)

(Hence Proved )

27.

Taking LHS

Material downloaded from myCBSEguide.com. 23 / 27


=

=
=
= = RHS

28.

Since and OP, OQ are radius through contact point must be on tangents lines.
Therefore, OPAQ is a square.
Let OP AP = AQ = OQ
Now in ABC ,

i.e. 36 + AC2 = 100

AC2 = 64 Ac = 8
BR = PB = 6 - x
CQ = 8 - x = CR
[ tangent from an external point ]

Now again BC = CR + BR
10 = 8 - x + 6 - x
10 = 14 - 2x 2x = 4
x = 2

Material downloaded from myCBSEguide.com. 24 / 27


Area of shaded portion = Area of triangle - Area of circle

29.

Radius of the hemispherical part = 1.4 cm


Length of the cylindrical part = 5 - (2 x 1.4) = 2.2cm
Volume of 1 gulab jamun = volume of two hemispherical ends + Volume of cylindrical part

Volume of 45 gulab jamuns


Volume of syrup found in 45 gulab jamuns

30. Solution : table for ‘less than Ogive’ and ‘more than Ogive’

For ‘less than’ Ogive For ‘more than’ Ogive


C.I.
C.I. (less than) c.f. Point C.I. (more than) c.f. Point

20- 30 10 30 10 (30,10) 20 100 (20,100)

30 - 40 8 40 18 (40,18) 30 90 (30,90)

40 - 50 12 50 30 (50,30) 40 82 (40,82)

50 60 24 60 54 (60,54) 50 70 (50,70)

Material downloaded from myCBSEguide.com. 25 / 27


60 - 70 6 70 60 (70,60) 60 46 (60,46)

70 - 80 25 80 85 (80,85) 70 40 (70,40)

80 - 90 15 90 100 (90,100) 80 15 (80,15)

‘Less than’ Ogive and ‘more than’ Ogive.

We notice both the curves intersect at (58.3, 50)


Median = 58.3

OR

We have,

class Mid- value(xi) (fi) f iu i c.f

5-15 10 2 -3 -6 2

15-25 20 3 -2 -6 5

Material downloaded from myCBSEguide.com. 26 / 27


25-35 30 5 -1 -5 10

35-45 40 7 0 0 17

45-55 50 4 1 4 21

55-65 60 2 2 4 23

65-75 70 2 3 6 25

Let assumed mean ‘a’ = 40, Here h=10

Mean

= 40 - 1.2 = 38.8
Since, maximum frequency = 7
Modal class = 35-45
Here, l = 35, f1 = 7, f0 = 5, f2 = 4

We know that

Mode

= 35 + 4 = 39
Since which lies in 35-45 class

Here,

Median =

= 35 + 3.6 nearly = 38.6 nearly

Material downloaded from myCBSEguide.com. 27 / 27

S-ar putea să vă placă și